++
++
++
++
All of the following hormones is produced by the anterior pituitary EXCEPT:
++
++
++
A. Adrenocorticotropic hormone
++
++
++
++
++
++
++
++
E. Thyroid-stimulating hormone
+
++
The answer is C. (Chap. 401e) Hormones produced by the anterior pituitary include adrenocorticotropic hormone (ACTH), thyroid-stimulating hormone (TSH), luteinizing hormone (LH), follicle-stimulating hormone (FSH), prolactin, and growth hormone. The posterior pituitary produces vasopressin and oxytocin. The anterior and posterior pituitary have a separate vascular supply, and the posterior pituitary is directly innervated by the hypothalamic neurons via the pituitary stalk, thus making it susceptible to shear stress–associated dysfunction. Hypothalamic control of anterior pituitary function is through secreted hormones; thus, it is less susceptible to traumatic injury.
++
++
++
A 45-year-old man reports to his primary care physician that his wife has noted coarsening of his facial features over several years. In addition, he reports low libido and decreased energy. Physical examination shows frontal bossing and enlarged hands. An MRI confirms that he has a pituitary mass. Which of the following screening tests should be ordered to diagnose the cause of the mass?
++
++
++
A. 24-Hour urinary free cortisol
++
++
B. Adrenocorticotropic hormone (ACTH) assay
++
++
++
++
D. Serum insulin-like growth factor-1 (IGF-1) level
++
++
+
++
The answer is D. (Chap. 401e) Functional pituitary adenoma presentations include acromegaly, as in this patient; prolactinomas; or Cushing syndrome. Hypersecretion of growth hormone underlies this syndrome in patients with pituitary masses, although ectopic production of growth hormone, particularly by tumors, has been reported. Because growth hormone is secreted in a highly pulsatile fashion, obtaining random serum levels is not reliable. Thus, the downstream mediator of systemic effects of growth hormone, insulin-like growth factor-1 (IGF-1), is measured to screen for growth hormone excess. IGF-1 is made by the liver in response to growth hormone stimulation. An oral glucose tolerance test with growth hormone obtained at 0, 30, and 60 minutes may also be used to screen for acromegaly because normal persons should suppress growth hormone to this challenge. Serum prolactin level is useful to screen for prolactinomas, and 24-hour urinary free cortisol and ACTH assay are useful screens for Cushing disease.
++
++
++
Which of the following statements regarding the anatomy of the pituitary gland is true?
++
++
++
A. Growth hormone is derived from the precursor proopiomelanocortin (POMC).
++
++
B. Prolactin-secreting cells form the majority of cells in the anterior pituitary.
++
++
C. The anterior pituitary secretes hormones directly synthesized in neuroendocrine cells in the hypothalamus.
++
++
D. The pituitary gland forms from the Rathke pouch embryonically.
++
++
E. The posterior pituitary has dual arterial blood supply.
+
++
The answer is D. (Chap. 401e) The pituitary gland does form from the Rathke pouch embryonically. As shown in Figure X-3, blood supply of the pituitary gland comes from the superior and inferior hypophyseal arteries. The hypothalamic pituitary portal plexus provides the major blood source for the anterior pituitary, allowing reliable transmission of hypothalamic peptide pulses without significant systemic dilution; consequently, pituitary cells are exposed to releasing or inhibiting factors and, in turn, release their hormones as discrete pulses into the systemic circulation. The posterior pituitary is supplied by the inferior hypophyseal arteries. In contrast to the anterior pituitary, the posterior lobe is directly innervated by hypothalamic neurons (supraopticohypophyseal and tuberohypophyseal nerve tracts) via the pituitary stalk. Thus, posterior pituitary production of vasopressin (antidiuretic hormone) and oxytocin is particularly sensitive to neuronal damage by lesions that affect the pituitary stalk or hypothalamus. ACTH is derived from proopiomelanocortin, and prolactin is secreted in the posterior pituitary.
+
++
++
+
++
++
++
A 58-year-old man undergoes severe head trauma and develops pituitary insufficiency. After recovery, he is placed on thyroid hormone, testosterone, glucocorticoids, and vasopressin. On a routine visit, he questions his primary care physician regarding potential growth hormone deficiency. All of the following are potential signs or symptoms of growth hormone deficiency EXCEPT:
++
++
++
A. Abnormal lipid profile
++
++
++
++
C. Increased bone mineral density
++
++
D. Increased waist-to-hip ratio
++
++
E. Left ventricular dysfunction
+
++
The answer is C. (Chap. 401e) Adult growth hormone deficiency is usually caused by hypothalamic or pituitary damage. Because growth hormone is no longer important for achieving stature, the presentation is different from childhood growth hormone deficiency. Although growth hormone has direct tissue effects, it primarily acts through increasing secretion of IGF-1, which in turn stimulates lipolysis, increases circulating fatty acids, reduces omental fat mass, and enhances lean body mass. Thus, deficiency of growth hormone causes the opposite effects. In addition, hypertension, left ventricular dysfunction, and increased plasma fibrinogen levels may also be present with deficient growth hormone. Reduced, not increased, bone mineral density may also occur in adults with growth hormone deficiency.
++
++
++
A 75-year-old man presents with development of abdominal obesity, proximal myopathy, and skin hyperpigmentation. His laboratory evaluation shows a hypokalemic metabolic alkalosis. Cushing syndrome is suspected. Which of the following statements regarding this syndrome is true?
++
++
++
A. Basal ACTH level is likely to be low.
++
++
B. Circulating corticotropin-releasing hormone is likely to be elevated.
++
++
C. Pituitary magnetic resonance imaging (MRI) will visualize all ACTH-secreting tumors.
++
++
D. Referral for urgent performance of inferior petrosal venous sampling is indicated.
++
++
E. Serum potassium level <3.3 mmol/L is suggestive of ectopic ACTH production.
+
++
The answer is E. (Chap. 401e) The patient has a clinical presentation consistent with Cushing syndrome. Although many cases of inappropriate elevation of ACTH are due to pituitary tumors, a substantial proportion are due to ectopic ACTH secretion. Clues to this diagnosis include a rapid onset of hypercortisolism features associated with skin hyperpigmentation and severe myopathy. Additionally, hypertension, hypokalemic metabolic alkalosis, glucose intolerance, and edema are more prominent in ectopic ACTH secretion than in pituitary tumors. Serum potassium <3.3 mmol/L is present in 70% of patients with ectopic ACTH, but <10% of patients with pituitary-dependent Cushing syndrome. ACTH levels will be high, because this is the underlying cause of both types of Cushing syndrome. Corticotropin-releasing hormone is rarely the cause of Cushing syndrome. Unfortunately, magnetic resonance imaging (MRI) of the pituitary gland will not visualize lesions <2 mm; thus occasionally, sampling of the inferior petrosal veins is required, but this is not yet indicated in this patient at this point in the evaluation.
++
++
++
Which of the following is common in patients with Kallmann syndrome?
++
++
++
++
++
++
++
C. Precocious (early) puberty in females
++
++
++
++
+
++
The answer is A. (Chap. 402) Kallmann syndrome results from defective hypothalamic gonadotropin-releasing hormone (GnRH) synthesis and is associated with anosmia or hyposmia due to olfactory bulb agenesis or hypoplasia. Classically, the syndrome may also be associated with color blindness, optic atrophy, nerve deafness, cleft palate, renal abnormalities, cryptorchidism, and neurologic abnormalities such as mirror movements. Associated clinical features, in addition to GnRH deficiency, vary depending on the genetic cause. GnRH deficiency prevents progression through puberty. Males present with delayed puberty and pronounced hypogonadal features, including micropenis, which are probably the result of low testosterone levels during infancy. Females present with primary amenorrhea and failure of secondary sexual development. A white forelock is typical of Waardenburg syndrome, whereas hyperphagic obesity is common in Prader-Willi syndrome.
++
++
++
A 22-year-old woman who is otherwise healthy undergoes an uneventful vaginal delivery of a full-term infant. One day postpartum, she complains of visual changes and severe headache. Two hours after these complaints, she is found unresponsive and profoundly hypotensive. She is intubated and placed on mechanical ventilation. Her blood pressure is 68/28 mmHg, regular heart rate is 148 bpm, her oxygen saturation is 95% on fraction of inspired oxygen (FiO2) of 0.40. Physical exam is unremarkable. Her laboratory tests are notable for glucose of 49 mg/dL, with normal hematocrit and white blood cell count. Which of the following is most likely to reverse her hypotension?
++
++
++
A. Activated drotrecogin alfa
++
++
++
++
C. Piperacillin/tazobactam
++
++
++
++
E. Transfusion of packed red blood cells
+
++
The answer is B. (Chap. 402) This patient has evidence of Sheehan syndrome after giving birth. In this syndrome, the postpartum hyperplastic pituitary is at increased risk for hemorrhage and/or infarction. This leads to bilateral visual changes, headache, and meningeal signs. Ophthalmoplegia may be observed. In severe cases, cardiovascular collapse and altered levels of consciousness may be observed. Laboratory evaluation commonly shows hypoglycemia. Pituitary computed tomography (CT) or MRI may show signs of sellar hemorrhage if present. Involvement of all pituitary hormones may be seen, although the most acute findings are often hypoglycemia and hypotension from failure of ACTH. The hypoglycemia and hypotension present in this patient suggest failure of the glucocorticoid system; thus treatment with a corticosteroid is indicated. There is no evidence of sepsis; thus antibiotics and drotrecogin alfa are not indicated. With a normal hematocrit and no reported evidence of massive hemorrhage, packed red cell transfusion is unlikely to be helpful. Although TSH production is undoubtedly low in this patient, the most immediate concern is replacement of glucocorticoid.
++
++
++
You are caring for Mr. Gelston, a 19-year-old man who had a brain tumor when young and underwent cranial radiation. You note that he has short stature and has not yet gone through puberty. You suspect that he has pituitary insufficiency due to radiation. Which of the following is true regarding acquired hypopituitarism due to radiation?
++
++
++
A. At a dose of 50 Gy of radiation, only 5% of patients will manifest hypopituitarism.
++
++
B. The majority of patients who develop hypopituitarism after cranial radiation, do so within a year of treatment.
++
++
C. Growth hormone is the most common hormonal deficiency.
++
++
D. There is no correlation between radiation dose and likelihood of developing hypopituitarism.
++
++
E. Older adults are at highest risk from radiation-induced hypopituitarism.
+
++
The answer is C. (Chap. 402) Cranial irradiation may result in long-term hypothalamic and pituitary dysfunction, especially in children and adolescents, because they are more susceptible to damage after whole-brain or head and neck therapeutic irradiation. The development of hormonal abnormalities correlates strongly with irradiation dosage and the time interval after completion of radiotherapy. Up to two-thirds of patients ultimately develop hormone insufficiency after a median dose of 50 Gy (5000 rad) directed at the skull base. The development of hypopituitarism occurs over 5–15 years and usually reflects hypothalamic damage rather than primary destruction of pituitary cells. Although the pattern of hormone loss is variable, growth hormone deficiency is most common, followed by gonadotropin and ACTH deficiency. When deficiency of one or more hormones is documented, the possibility of diminished reserve of other hormones is likely. Accordingly, anterior pituitary function should be continually evaluated over the long term in previously irradiated patients, and replacement therapy should be instituted when appropriate.
++
++
++
A 23-year-old college student is followed in the student health center for medical management of panhypopituitarism after resection of craniopharyngioma as a child. She reports moderate compliance with her medications but feels generally well. Thyroid-stimulating hormone (TSH) is checked and is below the limits of detection of the assay. Which of the following is the next most appropriate action?
++
++
++
A. Decrease levothyroxine dose to half of current dose
++
++
++
++
++
++
D. Order MRI of her brain
++
++
E. Order thyroid uptake scan
+
++
The answer is C. (Chap. 402) The patient has panhypopituitarism and is unable to make TSH; thus her plasma TSH level will always be low, regardless of the adequacy of her thyroxine (T4) replacement. A free T4 level will allow determination of whether her plasma level is in the normal range of thyroid hormone. This, coupled with her symptoms, will aid in determination of proper levothyroxine dosing. There is no evidence of recurrent disease clinically; thus MRI is not useful. She is unlikely to have primary thyroid disease, and T4 level is unknown presently, so thyroid uptake scan is not indicated at this time.
++
++
++
A patient visited a local emergency department 1 week ago with a headache. She received a head MRI, which did not reveal a cause for her symptoms, but the final report states “An empty sella is noted. Advise clinical correlation.” The patient was discharged from the emergency department with instructions to follow up with her primary care physician as soon as possible. Her headache has resolved, and the patient has no complaints. However, she comes to your office 1 day later very concerned about this unexpected MRI finding. What should be the next step in her management?
++
++
++
A. Diagnose her with subclinical panhypopituitarism, and initiate low-dose hormone replacement.
++
++
B. Reassure her and follow laboratory results closely.
++
++
C. Reassure her and repeat MRI in 6 months.
++
++
D. This may represent early endocrine malignancy, so whole-body positron emission tomography (PET)/computed tomography (CT) is indicated.
++
++
E. This MRI finding likely represents the presence of a benign adenoma, so she should be referred to neurosurgery for resection.
+
++
The answer is B. (Chap. 402) The identification of an empty sella is often an incidental MRI finding. Typically, these patients will have normal pituitary function and should be reassured. It is likely that the surrounding rim of pituitary tissue is functioning normally. An empty sella may signal the insidious onset of hypopituitarism, and laboratory results should be followed closely. Unless her clinical situation changes, repeat MRI is not indicated. Endocrine malignancy is unlikely, and surgery is not part of the management of an empty sella.
++
++
++
Pituitary adenomas typically expand in which direction?
++
++
++
++
++
++
++
++
++
++
++
+
++
The answer is E. (Chap. 403) The dorsal sellar diaphragm presents the least resistance to soft tissue expansion from the sella; consequently, pituitary adenomas frequently extend in a suprasellar direction. Bony invasion may occur as well.
++
++
++
On MRI of the pituitary, which of the following findings is abnormal in an adult?
++
++
++
A. A slightly concave upper aspect of the pituitary
++
++
B. Brighter T1 intensity of the posterior pituitary
++
++
C. Heterogeneous anterior pituitary tissue
++
++
D. Pituitary height of 8–12 mm
++
++
E. Tissue that is lower intensity than the nearby brain tissue on T1 images and enhances on T2 images
+
++
The answer is E. (Chap. 403) Pituitary gland height ranges from 6 mm in children to 8 mm in adults; during pregnancy and puberty, the height may reach 10–12 mm. The upper aspect of the adult pituitary is flat or slightly concave, but in adolescent and pregnant individuals, this surface may be convex, reflecting physiologic pituitary enlargement. The stalk should be midline and vertical. Anterior pituitary gland soft tissue consistency is slightly heterogeneous on MRI, and signal intensity resembles that of brain matter on T1-weighted imaging. Adenoma density is usually lower than that of surrounding normal tissue on T1-weighted imaging, and the signal intensity increases with T2-weighted images. The high phospholipid content of the posterior pituitary results in a pituitary “bright spot.”
++
++
++
Mr. Jones has a pituitary adenoma on imaging that has extended directly superiorly and is compressing his optic chiasm. Which of the following visual field deficits is most likely present?
++
++
++
A. Bilateral inferior visual field deficits
++
++
B. Bilateral superior visual field deficits
++
++
++
++
D. Central scotomas bilaterally
++
++
E. Right homonymous hemianopia
+
++
The answer is C. (Chap. 403) Because optic tracts may be contiguous to an expanding pituitary mass, reproducible visual field assessment using perimetry techniques should be performed on all patients with sellar mass lesions that impinge the optic chiasm. Bitemporal hemianopia, often more pronounced superiorly, is observed classically. It occurs because nasal ganglion cell fibers, which cross in the optic chiasm, are especially vulnerable to compression of the ventral optic chiasm. Occasionally, homonymous hemianopia occurs as a result of postchiasmal compression, or monocular temporal field loss occurs as a result of prechiasmal compression.
++
++
++
All of the following features are present in Carney syndrome EXCEPT:
++
++
++
++
++
++
++
++
++
D. Hypertrophic cardiomyopathy
++
++
E. Spotty skin pigmentation
+
++
The answer is D. (Chap. 403) Carney syndrome is characterized by spotty skin pigmentation, myxomas, and endocrine tumors, including testicular, adrenal, and pituitary adenomas. Acromegaly occurs in about 20% of these patients. A subset of patients have mutations in the R1α regulatory subunit of protein kinase A (PRKAR1A).
++
++
++
Which of the following is the most common cause of preventable mental deficiency in the world?
++
++
++
++
++
++
++
++
++
++
++
+
++
The answer is B. (Chap. 405) Nutritional and maternal iodine deficiencies are common in many parts of the developing world and, when severe, can result in cretinism. Cretinism is characterized by mental and growth retardation but is preventable by administration of iodine and/or thyroid hormone early in life. Concomitant selenium deficiency can contribute to the neurologic manifestations. Iodine supplementation of bread, salt, and other foods has markedly decreased the rates of this disease. Beriberi disease is a nervous system ailment caused by a thiamine deficiency in the diet. Scurvy is due to vitamin C deficiency. Folate deficiency in pregnant women is associated with an increased risk of preterm labor and a number of congenital malformations, most notably involving the neural tube. Folate supplementation can lower the risk of spina bifida, anencephaly, congenital heart disease, cleft lips, and limb deformities. Vitamin A deficiency is a common cause of blindness in the developing world.
++
++
++
All of the following are associated with increased levels of total T4 in the plasma with a normal free T4 EXCEPT:
++
++
++
++
++
++
++
C. Euthyroid sick syndrome
++
++
D. Familial dysalbuminemic hyperthyroxinemia
++
++
E. Familial excess thyroid binding globulin
+
++
The answer is C. (Chap. 405) A number of conditions are associated with normal thyroid function but hyperthyroxinemia. Although some of these are associated with clinical hyperthyroidism, many have simply elevated levels of total T4 and normal conversion to triiodothyronine (T3) and, thus, are clinically normal. Anything that increases liver production of thyroid-binding globulin will produce elevated total T4 levels and normal free T4 and T3 levels. In this category are pregnancy, estrogen-containing oral contraceptives, cirrhosis, and familial excess thyroid-binding globulin production. Familial dysalbuminemic hyperthyroxinemia results in an albumin mutation and increased T4 with normal free T4 and T3 levels. Euthyroid sick syndrome occurs during acute medical and psychiatric illness. In this syndrome, there is transiently increased unbound T4 and decreased TSH. Total T4 and T3 may be decreased, particularly later in the course of disease.
++
++
++
Which of the following is the most common cause of hypothyroidism worldwide?
++
++
++
++
++
++
++
C. Iatrogenic hypothyroidism
++
++
++
++
+
++
The answer is D. (Chap. 405) Iodine deficiency remains the most common cause of hypothyroidism worldwide. It is present at relatively high levels even in the developed world, including Europe. In areas of iodine sufficiency, autoimmune disease (Hashimoto thyroiditis) and iatrogenic hypothyroidism (treatment of hyperthyroidism) are the most common causes.
++
++
++
A 75-year-old woman is diagnosed with hypothyroidism. She has longstanding coronary artery disease and is wondering about the potential consequences for her cardiovascular system. Which of the following statements is true regarding the interaction of hypothyroidism and the cardiovascular system?
++
++
++
A. A reduced stroke volume is found with hypothyroidism.
++
++
B. Blood flow is diverted toward the skin in hypothyroidism.
++
++
C. Myocardial contractility is increased with hypothyroidism.
++
++
D. Pericardial effusions are a rare manifestation of hypothyroidism.
++
++
E. Reduced peripheral resistance is found in hypothyroidism and may be accompanied by hypotension.
+
++
The answer is A. (Chap. 405) There are a number of important effects of thyroid hormone (or its absence) on the cardiovascular system. Importantly, hypothyroidism is associated with bradycardia, reduced myocardial contractility, and thus reduced stroke volume. Increased peripheral resistance may be accompanied by systemic hypertension, particularly diastolic in hypothyroidism. Pericardial effusions are found in up to 30% of patients with hypothyroidism, although they rarely cause decreased cardiac function. Finally, in hypothyroid patients, blood flow is directed away from the skin, thus producing cool extremities.
++
++
++
A 38-year-old mother of three presents to her primary care office with complaints of fatigue. She feels that her energy level has been low for the past 3 months. She was previously healthy and taking no medications. She does report that she has gained about 10 lb and has severe constipation, for which she has been taking a number of laxatives. A TSH is elevated at 25 mU/L. Free T4 is low. She is wondering why she has hypothyroidism. Which of the following tests is most likely to diagnose the etiology?
++
++
++
A. Antithyroid peroxidase antibody
++
++
B. Antithyroglobulin antibody
++
++
C. Radioiodine uptake scan
++
++
D. Serum thyroglobulin level
++
++
+
++
The answer is A. (Chap. 405) The most common cause of hypothyroidism in the United States is autoimmune thyroiditis, because the United States is an iodine-replete area. Although earlier in the disease, a radioiodine uptake scan may have shown diffusely increased uptake from lymphocytic infiltration, at this point in the disease, when the infiltrate is “burned out,” there is likely to be little found on the scan. Likewise, a thyroid ultrasound would only be useful for presumed multinodular goiter. Antithyroid peroxidase antibodies are commonly found in patients with autoimmune thyroiditis, whereas antithyroglobulin antibodies are found less commonly. Antithyroglobulin antibodies are also found in other thyroid disorders (Graves disease, thyrotoxicosis) as well as systemic autoimmune diseases (systemic lupus erythematosus). Thyroglobulin is released from the thyroid in all types of thyrotoxicosis with the exception of factitious disease. This patient, however, was hypothyroid, and thus serum thyroglobulin levels are unlikely to be helpful.
++
++
++
A 54-year-old woman with longstanding hypothyroidism is seen in her primary care physician’s office for a routine evaluation. She reports feeling fatigued and somewhat constipated. Since her last visit, her other medical conditions, which include hypercholesterolemia and systemic hypertension, have been stable. She was diagnosed with uterine fibroids and started on iron recently. Her other medications include levothyroxine, atorvastatin, and hydrochlorothiazide. A TSH is checked, and it is elevated to 15 mU/L. Which of the following is the most likely reason for her elevated TSH?
++
++
++
++
++
++
++
C. Medication noncompliance
++
++
D. Poor absorption of levothyroxine due to ferrous sulfate
++
++
E. TSH-secreting pituitary adenoma
+
++
The answer is D. (Chap. 405) An increase in TSH in a patient with hypothyroidism that was previously stable with medication for many years suggests either a failure of taking the medication, difficulty with absorption from bowel disease, or medication interaction or drug–drug interaction affecting clearance. In patients with normal body weight taking >200 μg of levothyroxine per day, an elevated TSH strongly suggests noncompliance. Such patients should be encouraged to take two tablets at one time on the day they remember to attempt to reach the weekly target dose; the long drug half-life makes this practice safe. Other causes of increased thyroxine requirements include malabsorption, such as with celiac disease or small bowel surgery, estrogen therapy, and drugs that interfere with T4 absorption (e.g., ferrous sulfate, cholestyramine) or clearance (e.g., lovastatin, amiodarone, carbamazepine, phenytoin).
++
++
++
An 87-year-old woman is admitted to the intensive care unit with depressed level of consciousness, hypothermia, sinus bradycardia, hypotension, and hypoglycemia. She was previously healthy with the exception of hypothyroidism and systemic hypertension. Her family recently checked in on her and found that she was not taking any of her medications because of financial difficulties. There is no evidence of infection on exam, urine microscopy, or chest radiograph. Her serum chemistries are notable for mild hyponatremia and a glucose of 48 mg/dL. A TSH is >100 mU/L. All of the following statements regarding this condition are true EXCEPT:
++
++
++
A. External warming is a critical feature of therapy in patients with a temperature <34°C.
++
++
B. Hypotonic intravenous solutions should be avoided.
++
++
C. Intravenous (IV) levothyroxine should be administered with IV glucocorticoids.
++
++
D. Sedation should be avoided if possible.
++
++
E. This condition occurs almost exclusively in the elderly and often is precipitated by an unrelated medical illness.
+
++
The answer is A. (Chap. 405) The patient has myxedema coma. This condition of profound hypothyroidism most commonly occurs in the elderly, and often a precipitating condition may be identified such as myocardial infarction or infection. Clinical manifestations include altered level of consciousness, bradycardia, and hypothermia. Management includes repletion of thyroid hormone through IV levothyroxine, but also supplementation of glucocorticoids because there is impaired adrenal reserve in severe hypothyroidism. Care must be taken with rewarming because it may precipitate cardiovascular collapse. Therefore, external warming is indicated only if the temperature is <30°C. Hypertonic saline and glucose may be used if hyponatremia or hypoglycemia is severe; however, hypotonic solutions should be avoided because they may worsen fluid retention. Because metabolism of many substances is markedly reduced, sedation should be avoided or minimized. Similarly, blood levels of drugs should be monitored when available.
++
++
++
A 29-year-old woman is evaluated for anxiety, palpitations, and diarrhea and found to have Graves disease. Before she begins therapy for her thyroid condition, she has an episode of acute chest pain and presents to the emergency department. Although a CT angiogram is ordered, the radiologist calls to notify the treating physician that this is potentially dangerous. Which of the following best explains the radiologist’s recommendation?
++
++
++
A. Iodinated contrast exposure in patients with Graves disease may exacerbate hyperthyroidism.
++
++
B. Pulmonary embolism is exceedingly rare in Graves disease.
++
++
C. Radiation exposure in patients with hyperthyroidism is associated with increased risk of subsequent malignancy.
++
++
D. Tachycardia with Graves disease limits the image quality of CT angiography and will not allow accurate assessment of pulmonary embolism.
++
++
E. The radiologist was mistaken; CT angiography is safe in Graves disease.
+
++
The answer is A. (Chap. 405) Patients with Graves disease produce thyroid-stimulating immunoglobulins. They subsequently produce higher levels of T4 compared with the normal population. As a result, many patients with Graves disease are mildly iodine deficient, and T4 production is somewhat limited by the availability of iodine. Exposure to iodinated contrast thus reverses iodine deficiency and may precipitate worsening hyperthyroidism. Additionally, the reversal of mild iodine deficiency may make iodine-125 therapy for Graves disease less successful because thyroid iodine uptake is lessened in the iodine-replete state.
++
++
++
A patient has neurosurgery for a pituitary tumor that requires resection of the gland. Which of the following functions of the adrenal gland will be preserved in this patient immediately postoperatively?
++
++
++
A. Morning peak of plasma cortisol level
++
++
B. Release of cortisol in response to stress
++
++
C. Sodium retention in response to hypovolemia
++
++
+
++
The answer is C. (Chap. 406) The adrenal gland has three major functions: glucocorticoid synthesis, aldosterone synthesis, and androgen precursor synthesis. Glucocorticoid synthesis is controlled by the pituitary secretion of ACTH. The primary stimulus for aldosterone synthesis is the renin-angiotensin-aldosterone system, which is independent of the pituitary. Thus, morning cortisol secretion and release of cortisol in response to stress are regulated by the pituitary gland, whereas regulation of sodium retention and potassium excretion by aldosterone is independent of the pituitary and would be preserved in this patient.
++
++
++
Which of the following is the most common cause of Cushing syndrome?
++
++
++
A. ACTH-producing pituitary adenoma
++
++
B. Adrenocortical adenoma
++
++
C. Adrenocortical carcinoma
++
++
D. Ectopic ACTH secretion
++
++
E. McCune-Albright syndrome
+
++
The answer is A. (Chap. 406) Cushing syndrome is a constellation of features that result from chronic exposure to elevated levels of cortisol from any etiology. Although the most common etiology is an ACTH-producing pituitary adenoma, which accounts for 75% of Cushing syndrome, 15% of cases are due to ectopic ACTH syndromes such as bronchial or pancreatic tumors, small-cell lung cancer, and other causes. ACTH-independent Cushing syndrome is much rarer. Adrenocortical adenoma underlies 5%–10% of cases, and adrenocortical carcinoma is present in 1% of Cushing cases. McCune-Albright syndrome is a genetic cause of bone abnormalities, skin lesions (café-au-lait), and premature puberty, particularly in girls. Interestingly, it is caused by a sporadic in utero mutation, not an inherited disorder, and thus will not be passed on to progeny.
++
++
++
All of the following are features of Conn syndrome EXCEPT:
++
++
++
++
++
++
++
++
++
++
++
E. Severe systemic hypertension
+
++
The answer is B. (Chap. 406) Conn syndrome refers to an aldosterone-producing adrenal adenoma. Although it accounts for 40% of hyperaldosterone states, bilateral micronodular adrenal hyperplasia is more common. Other causes of hyperaldosteronism are substantially rarer, accounting for <1% of disease. The hallmark of Conn syndrome is hypertension with hypokalemia. Because aldosterone stimulates sodium retention and potassium excretion, all patients should be hypokalemic at presentation. Serum sodium is usually normal because of concurrent fluid retention. Hypokalemia may be associated with muscle weakness, proximal myopathy, or even paralysis. Hypokalemia may be exacerbated by thiazide diuretics. Additional features include metabolic alkalosis that may contribute to muscle cramps and tetany.
++
++
++
All of the following statements regarding asymptomatic adrenal masses (incidentalomas) are true EXCEPT:
++
++
++
A. All patients with incidentalomas should be screened for pheochromocytoma.
++
++
B. Fine-needle aspiration may distinguish between benign and malignant primary adrenal tumors.
++
++
C. In patients with a history of malignancy, the likelihood that the adrenal mass is a metastasis is approximately 50%.
++
++
D. The majority of adrenal incidentalomas are nonsecretory.
++
++
E. The vast majority of adrenal incidentalomas are benign.
+
++
The answer is B. (Chap. 406) Incidental adrenal masses are often discovered during radiographic testing for another condition and are found in approximately 6% of adults at autopsy. Fifty percent of patients with a history of malignancy and a newly discovered adrenal mass will actually have an adrenal metastasis. Fine-needle aspiration of a suspected metastatic malignancy will often be diagnostic. In the absence of a suspected nonadrenal malignancy, most adrenal incidentalomas are benign. Primary adrenal malignancies are uncommon (<0.01%), and fine-needle aspiration is not useful to distinguish between benign and malignant primary adrenal tumors. Although 90% of these masses are nonsecretory, patients with an incidentaloma should be screened for pheochromocytoma and hypercortisolism with plasma free metanephrines and an overnight dexamethasone suppression test, respectively. When radiographic features suggest a benign neoplasm (<3 cm), scanning should be repeated in 3–6 months. When masses are >6 cm, surgical removal (if more likely primary adrenal malignancy) or fine-needle aspiration (if more likely metastatic malignancy) is preferred.
++
++
++
You are designing an experiment to determine the effect of psychosocial stress exposure on peak daily cortisol secretion. When should you measure cortisol to ensure that you are most likely assessing peak cortisol levels?
++
++
++
++
++
++
++
++
++
++
++
+
++
The answer is C. (Chap. 406) The release of corticotropin-releasing hormone, and subsequently ACTH, occurs in a pulsatile fashion that follows a circadian rhythm under the control of the hypothalamus, specifically its suprachiasmatic nucleus (SCN), with additional regulation by a complex network of cell-specific clock genes. Reflecting the pattern of ACTH secretion, adrenal cortisol secretion exhibits a distinct circadian rhythm; it starts to rise in the early morning hours prior to awakening, with peak levels in the morning and low levels in the evening (Figure X-27).
+
++
++
FIGURE X-27 Modified after Debono M, Ghobadi C, Rostami-Hodjegan A, et al: Modified-release hydrocortisone to provide circadian cortisol profiles. J Clin Endocrinol Metab 94:1548, 2009.
+
++
++
++
Mr. McTrap is admitted to the hospital after a car accident. His medical history is unknown, and on presentation, he is obtunded and can provide no history. CT scan reveals a splenic laceration, and he is emergently taken to the operating room for splenectomy, which proceeds without complication. At the completion of the operation, all bleeding has stopped, and he returns to the intensive care unit. However, he remains deeply hypotensive with a blood pressure of 70/50 mmHg with an increase only to 82/52 mmHg after a bolus of 2 L of normal saline IV. He is afebrile with a normal white blood cell count. Repeat CT scan of the chest, abdomen, and pelvis shows no hemorrhage. Jugular venous pressure is not visible above the clavicle. He has a round face and is obese, and you note the following on physical exam (see Figure X-28).
+
++
++
+
++
He has no hand hyperpigmentation. What is the next most appropriate step?
++
++
++
A. Return to the operating room for exploratory laparotomy
++
++
B. Administer hydrocortisone 100 mg IV
++
++
C. Administer vancomycin and piperacillin/tazobactam
++
++
D. Insert intra-aortic balloon pump for counterpulsation
++
++
E. Perform MRI of the spine
+
++
The answer is B. (Chap. 406) This patient is obese and has abdominal stria and a round (or moon) facies, which are all signs of glucocorticoid excess. Often, this is due to exogenous (corticosteroid) administration, although it could also be due to endogenous production (Cushing syndrome). A physiologic stressor, such as trauma or infection, may trigger adrenal crisis. Importantly (although not present in this case), hyperthyroidism can also trigger adrenal crisis via increased glucocorticoid inactivation. Thus, glucocorticoids must always be provided first in the setting of concomitant thyroid and adrenal insufficiency. Acute adrenal insufficiency requires immediate initiation of rehydration, usually carried out by saline infusion at initial rates of 1 L/hr with continuous cardiac monitoring. Glucocorticoid replacement should be initiated by bolus injection of 100 mg of hydrocortisone, followed by the administration of 100–200 mg of hydrocortisone over 24 hours, either by continuous infusion or by bolus intravenous (IV) or intramuscular injections. Mineralocorticoid replacement can be initiated once the daily hydrocortisone dose has been reduced to <50 mg because, at higher doses, hydrocortisone provides sufficient stimulation of mineralocorticoid receptors.
++
++
++
A 43-year-old man with episodic, severe hypertension is referred for evaluation of possible secondary causes of hypertension. He reports feeling well generally, except for episodes of anxiety, palpitations, and tachycardia with elevation in his blood pressure during these episodes. Exercise often brings on these events. The patient also has mild depression and is presently taking sertraline, labetalol, amlodipine, and lisinopril to control his blood pressure. Urine 24-hour total metanephrines are ordered and show an elevation of 1.5 times the upper limit of normal. Which of the following is the next most appropriate step?
++
++
++
A. Hold labetalol for 1 week and repeat testing
++
++
B. Hold sertraline for 1 week and repeat testing
++
++
C. Refer immediately for surgical evaluation
++
++
D. Measure 24-hour urine vanillylmandelic acid level
++
++
E. Obtain MRI of the abdomen
+
++
The answer is A. (Chap. 407) When the diagnosis of pheochromocytoma is entertained, the first step is measurement of catecholamines and/or metanephrines. This can be achieved by urinary tests for vanillylmandelic acid, catecholamines, fractionated metanephrines, or total metanephrines. Testing for total metanephrines has a high sensitivity and, therefore, is frequently used. A value of three times the upper limit of normal is highly suggestive of pheochromocytoma. Borderline elevations, as this patient had, are likely to be false positives. The next most appropriate step is to remove potentially confounding dietary or drug exposures, if possible, and repeat the test. Likely culprit drugs include levodopa, sympathomimetics, diuretics, tricyclic antidepressants, and α- and β-blockers (labetalol in this case). Sertraline is a selective serotonin reuptake inhibitor antidepressant, not a tricyclic. Alternatively, a clonidine suppression test may be ordered.
++
++
++
A 45-year-old man is diagnosed with pheochromocytoma after presentation with confusion, marked hypertension to 250/140 mmHg, tachycardia, headaches, and flushing. His fractionated plasma metanephrines show a normetanephrine level of 560 pg/mL and a metanephrine level of 198 pg/mL (normal values: normetanephrine, 18–111 pg/mL; metanephrine, 12–60 pg/mL). CT scanning of the abdomen with IV contrast demonstrates a 3-cm mass in the right adrenal gland. A brain MRI with gadolinium shows edema of the white matter near the parieto-occipital junction consistent with reversible posterior leukoencephalopathy. You are asked to consult regarding management. Which of the following statements is true regarding management of pheochromocytoma in this individual?
++
++
++
A. β-Blockade is absolutely contraindicated for tachycardia even after adequate α-blockade has been attained.
++
++
B. Immediate surgical removal of the mass is indicated, because the patient presented with hypertensive crisis with encephalopathy.
++
++
C. Salt and fluid intake should be restricted to prevent further exacerbation of the patient’s hypertension.
++
++
D. Treatment with phenoxybenzamine should be started at a high dose (20–30 mg three times daily) to rapidly control blood pressure, and surgery can be undertaken within 24–48 hours.
++
++
E. IV phentolamine is indicated for treatment of the hypertensive crisis. Phenoxybenzamine should be started at a low dose and titrated to the maximum-tolerated dose over 2–3 weeks. Surgery should not be planned until the blood pressure is consistently below 160/100 mmHg.
+
++
The answer is E. (Chap. 407) Complete removal of the pheochromocytoma is the only therapy that leads to a long-term cure, although 90% of tumors are benign. However, preoperative control of hypertension is necessary to prevent surgical complications and lower mortality. This patient is presenting with encephalopathy in a hypertensive crisis. The hypertension should be managed initially with IV medications to lower the mean arterial pressure by approximately 20% over the initial 24-hour period. Medications that can be used for hypertensive crisis in pheochromocytoma include nitroprusside, nicardipine, and phentolamine. Once the acute hypertensive crisis has resolved, transition to oral α-adrenergic blockers is indicated. Phenoxybenzamine is the most commonly used drug and is started at low doses (5–10 mg three times daily) and titrated to the maximum-tolerated dose (usually 20–30 mg daily). Once α-blockers have been initiated, β-blockade can be safely used and is particularly indicated for ongoing tachycardia. Liberal salt and fluid intake helps expand plasma volume and treat orthostatic hypotension. Once blood pressure is maintained below 160/100 mmHg with moderate orthostasis, it is safe to proceed to surgery. If blood pressure remains elevated despite treatment with α-blockade, the addition of calcium channel blockers, angiotensin receptor blockers, or angiotensin-converting enzyme inhibitors should be considered. Diuretics should be avoided because they will exacerbate orthostasis.
++
++
++
Mr. Robinson returns for a follow-up after a long hospital stay for hypertension when he was diagnosed with a pheochromocytoma and ultimately underwent a left adrenalectomy. He reports feeling well since then, and his hypertension is well controlled. He is curious about whether his pheochromocytoma was considered malignant. What should you tell him?
++
++
++
A. Approximately 50% of pheochromocytomas are malignant.
++
++
B. Cellular atypia and invasion of blood vessels on pathology define malignancy for pheochromocytoma.
++
++
C. 23I-metaiodobenzylguanidine scans are not useful in locating distant metastases.
++
++
D. The absence of distant metastases rules out malignant disease.
+
++
The answer is D. (Chap. 407) The diagnosis of malignant pheochromocytoma is problematic. The typical histologic criteria of cellular atypia, presence of mitoses, and invasion of vessels or adjacent tissues are insufficient for the diagnosis of malignancy in pheochromocytoma. Thus, the term malignant pheochromocytoma is restricted to tumors with distant metastases, most commonly found by nuclear medicine imaging in lungs, bone, or liver—locations suggesting a vascular pathway of spread.
++
++
++
An 18-year-old woman is evaluated at her primary care physician’s office for a routine physical. She is presently healthy. Her family history is notable for a father and two aunts with multiple endocrine neoplasia type 1 (MEN1), and the patient has undergone genetic testing and carries the MEN1 gene. Which of the following is the first and most common presentation for individuals with this genetic mutation?
++
++
++
++
++
++
++
++
++
++
++
E. Uncontrolled systemic hypertension
+
++
The answer is B. (Chap. 408) Multiple endocrine neoplasia (MEN) syndrome is defined as a disorder with neoplasms affecting two or more hormonal tissues in several members of the family. The most common of these is MEN type 1 (MEN1), which is caused by the gene coding the nuclear protein called Menin. MEN1 is associated with tumors or hyperplasia of the parathyroid, pancreas, pituitary, adrenal cortex, and foregut and/or subcutaneous or visceral lipomas. The most common and earliest manifestation is hyperparathyroidism with symptomatic hypercalcemia. This most commonly occurs in the late teenage years, and 93%–100% of mutation carriers develop this complication. Gastrinomas, insulinomas, and prolactinomas are less common and tend to occur in the 20s, 30s, and 40s. Pheochromocytoma may occur in MEN1 but is more commonly found in MEN type 2A (MEN2A) or von Hippel-Lindau syndrome.
++
++
++
You are seeing Mr. Avendaw in clinic today. He is a 35-year-old man who last year had a partial thyroidectomy for medullary thyroid carcinoma. You note that he was recently in the hospital and diagnosed with a pheochromocytoma, and after 2 weeks of intensive medical therapy, he underwent unilateral adrenalectomy. He is recovering nicely. You are reviewing his chart before the visit, when you note that on the pathology from his thyroid surgery last year, a single parathyroid gland was removed that was shown to be a parathyroid tumor. When you meet with Mr. Avendaw, you will tell him which of the following?
++
++
++
A. “Family and genetic screening for similar cancers is not useful because the mutations causing these cancers are certainly unrelated and spontaneously arise.”
++
++
B. “I suspect you have a syndrome called multiple endocrine neoplasia type 1.”
++
++
C. “I suspect you have a syndrome called multiple endocrine neoplasia type 2.”
++
++
D. “The partial thyroidectomy was an appropriate treatment for this condition.”
++
++
E. “These tumors were likely caused by a mutation in the Menin gene.”
+
++
The answer is C. (Chap. 408) MEN type 2 (MEN2), also called Sipple syndrome, is characterized by the association of medullary thyroid carcinoma (MTC), pheochromocytomas, and parathyroid tumors. In MEN2A (the most common variant), MTC is associated with pheochromocytomas in 50% of patients (may be bilateral) and with parathyroid tumors in 20% of patients. MEN1, which is also referred to as Wermer syndrome, is characterized by the triad of tumors involving the parathyroids, pancreatic islets, and anterior pituitary. MEN1 syndrome is caused by a mutation in the Menin (or MEN1) gene. MEN2 is caused by a mutation in the RET gene. Family and genetic screening both have high value in this syndrome (MEN2) because prophylactic thyroidectomy, with life-long thyroxine replacement, has dramatically improved outcomes in patients with MEN2 and MEN3, such that approximately 90% of young patients with RET mutations who had a prophylactic thyroidectomy have no evidence of persistent or recurrent MTC at 7 years after surgery. Partial thyroidectomy is inappropriate for this patient; in patients with clinically evident MTC, a total thyroidectomy with bilateral central resection is recommended.
++
++
++
Johnny Stewart, a 4-year-old boy, presents to the hospital with hypotension, lethargy, and hyponatremia. You also note that his potassium is elevated to 5.7 mEq/dL. He is afebrile and has a normal complete blood count. However, you note extensive oral thrush. On review of his chart, you note that he has had multiple treatment courses for thrush and cutaneous candidal infections. Human immunodeficiency virus (HIV) antibody tests have been negative. Also, you note that at 1 year of age, he had an episode of tetany prompting an emergent presentation to the hospital, where he was found to be hypocalcemic. Ultimately, he was diagnosed with hypoparathyroidism. Given his current presentation, which of the following is the most appropriate course of treatment?
++
++
++
++
++
++
++
C. Kayexalate, IV insulin, and albuterol to treat presumed hyperkalemic periodic paralysis
++
++
++
++
E. Urgent echocardiogram for suspected cardiac tamponade
+
++
The answer is B. (Chap. 409) This patient almost certainly is hypotensive, hyponatremic, and hyperkalemic from primary adrenal insufficiency. Given the concomitant presence of hypoparathyroidism and mucocutaneous candidiasis, he likely suffers from autoimmune polyendocrine syndrome (APS) type 1. Mucocutaneous candidiasis, hypoparathyroidism, and Addison disease form the three major components of this disorder. It is an autosomal recessive disorder caused by mutations in the AIRE gene (autoimmune regulator gene) found on chromosome 21. APS type 1 develops very early in life, often in infancy. Chronic mucocutaneous candidiasis without signs of systemic disease is often the first manifestation. Hypoparathyroidism usually develops next, followed by adrenal insufficiency. Regarding the treatment of adrenal crisis, several issues merit mention. Adrenal insufficiency can be masked by primary hypothyroidism by prolonging the half-life of cortisol. Therefore, the caveat is that replacement therapy with thyroid hormone can precipitate an adrenal crisis in an undiagnosed individual. Hence, all patients with hypothyroidism and the possibility of APS should be screened for adrenal insufficiency to allow treatment with glucocorticoids prior to the initiation of thyroid hormone replacement. Treatment of mucocutaneous candidiasis with ketoconazole in an individual with subclinical adrenal insufficiency may also precipitate adrenal crisis. This patient may have concurrent hypocalcemia that merits treatment in conjunction with the adrenal insufficiency.
++
++
++
Mr. David presents to the emergency department with numbness and weakness in his legs and feet. On exam, you find that he is numb to the knees and has marked weakness in ankle dorsiflexion and plantar flexion. Two years ago, he developed diabetes, and last year, he was admitted when found to be profoundly hypothyroid. On examination, he has hepatosplenomegaly and appears to have a dark tan despite having no sun exposure recently. Which of the following tests will likely help make his diagnosis?
++
++
++
A. Anti–nuclear antibody titer measurement
++
++
B. Anti–thymoglobulin antibody titer measurement
++
++
++
++
D. Serum protein electrophoresis
++
++
E. Skin biopsy searching for intravascular clonal T cells
+
++
The answer is D. (Chap. 409) This patient likely has POEMS (polyneuropathy, organomegaly, endocrinopathy, M-protein, and skin changes). Patients usually present with a progressive sensorimotor polyneuropathy, diabetes mellitus (50%), primary gonadal failure (70%), and a plasma cell dyscrasia with sclerotic bony lesions. Associated findings can be hepatosplenomegaly, lymphadenopathy, and hyperpigmentation. Patients often present in the fifth and sixth decades of life and have a median survival after diagnosis of less than 3 years. The detection of an M-protein on serum electrophoresis would make POEMS the most likely diagnosis.
++
++
++
A 37-year-old man is evaluated for infertility. He and his wife have been attempting to conceive a child for the past 2 years without success. He initially saw an infertility specialist but was referred to endocrinology after sperm analysis showed no sperm. He is otherwise healthy and only takes a multivitamin. On physical examination, his vital signs are normal. He is tall and has small testes, gynecomastia, and minimal facial and axillary hair. Chromosomal analysis confirms Klinefelter syndrome. Which of the following statements is true?
++
++
++
A. Androgen supplementation is of little use in this condition.
++
++
B. He is not at increased risk for breast tumors.
++
++
C. Increased plasma concentrations of estrogen are present.
++
++
D. Most cases are diagnosed before puberty.
++
++
E. Plasma concentrations of follicle-stimulating hormone (FSH) and luteinizing hormone (LH) are decreased in this condition.
+
++
The answer is C. (Chap. 410) Klinefelter syndrome is a chromosomal disorder with 47, XXY. Because the primary feature of this disorder is gonadal failure, low testosterone is present, and thus, increased LH and FSH are produced in an attempt to increase testosterone production in the feedback loop of sex hormones. Increased estrogen production is often present because of chronic Leydig cell stimulation by LH and aromatization of androstenedione by adipose tissue. The lower testosterone-to-estrogen ratio results in mild feminization with gynecomastia. Features of low testosterone are small testes and eunuchoid proportions with long legs and incomplete virilization. Biopsy of the testes, although rarely performed, shows hyalinization of the seminiferous tubules and azoospermia. Although severe cases are diagnosed prepubertally as a result of small testes and impaired androgenization, approximately 75% of cases are not diagnosed, and the frequency in the general population is 1 in 1000. Patients with Klinefelter syndrome are at increased risk of breast tumors, thromboembolic disease, learning difficulties, obesity, diabetes mellitus, and varicose veins.
++
++
++
A 17-year-old teenager is evaluated in your office for primary amenorrhea. She does not feel as if she has entered puberty because she has never had a menstrual period and has sparse axillary and pubic hair growth. On examination, she is noted to be 150 cm tall. She has a low hairline and slight webbing of her neck. Her FSH level is 75 mIU/mL, LH is 20 mIU/mL, and estradiol level is 2 pg/mL. You suspect Turner syndrome. All of the following tests are indicated in this individual EXCEPT:
++
++
++
A. Buccal smear for nuclear heterochromatin (Barr body)
++
++
++
++
++
++
++
++
+
++
The answer is A. (Chap. 410) Turner syndrome most frequently results from a 45,X karyotype, but mosaicism (45,X/46,XX) also can result in this disorder. Clinically, Turner syndrome manifests as short stature and primary amenorrhea if presenting in young adulthood. In addition, chronic lymphedema of the hands and feet, nuchal folds, a low hairline, and high-arched palate are also common features. To diagnose Turner syndrome, karyotype analysis should be performed. A Barr body results from inactivation of one of the X chromosomes in women and is not seen in males. In Turner syndrome, the Barr body should be absent, but only 50% of individuals with Turner syndrome have the 45,X karyotype. Thus, the diagnosis could be missed in those with mosaicism or other structural abnormalities of the X chromosome. Multiple comorbid conditions are found in individuals with Turner syndrome, and appropriate screening is recommended. Congenital heart defects affect 30% of women with Turner syndrome, including bicuspid aortic valve, coarctation of the aorta, and aortic root dilatation. An echocardiogram should be performed, and the individual should be assessed with blood pressures in the arms and legs. Hypertension can also be associated with structural abnormalities of the kidney and urinary tract, most commonly horseshoe kidney. A renal ultrasound is also recommended. Autoimmune thyroid disease affects 15%–30% of women with Turner syndrome and should be assessed by screening TSH. Other comorbidities that may occur include sensorineural hearing loss, elevated liver function enzymes, osteoporosis, and celiac disease.
++
++
++
An infant is born with ambiguous genitalia. Although amniocentesis analysis during pregnancy showed a 46, XX genotype, this infant has phallic-appearing genitalia and partially fused labia. You cannot palpate testes. Aside from a standard blood test, which biochemical screen is indicated?
++
++
++
A. Flow cytometry of the peripheral blood
++
++
++
++
C. Serum 17-hydroxyprogesterone levels
++
++
++
++
E. Serum prolactin levels
+
++
The answer is C. (Chap. 410) This infant likely has congenital adrenal hyperplasia (CAH). The classic form of 21-hydroxylase deficiency (21-OHD) is the most common cause of CAH. It has an incidence between 1 in 10,000 and 1 in 15,000 and is the most common cause of androgenization in chromosomal 46,XX females. Affected individuals are homozygous or compound heterozygous for severe mutations in the enzyme 21-hydroxylase (CYP21A2). This mutation causes a block in adrenal glucocorticoid and mineralocorticoid synthesis, increasing 17-hydroxyprogesterone and shunting steroid precursors into the androgen synthesis pathway. Glucocorticoid insufficiency causes a compensatory elevation of ACTH, resulting in adrenal hyperplasia and additional synthesis of steroid precursors proximal to the enzymatic block. Increased androgen synthesis in utero causes androgenization of the 46,XX fetus in the first trimester. Ambiguous genitalia are seen at birth, with varying degrees of clitoral enlargement and labial fusion. The salt-wasting form of 21-OHD results from severe combined glucocorticoid and mineralocorticoid deficiency. A salt-wasting crisis usually manifests between 5 and 21 days of life and is a potentially life-threatening event that requires urgent fluid resuscitation and steroid treatment. Thus, a diagnosis of 21-OHD should be considered in any baby with atypical genitalia with bilateral nonpalpable gonads.
++
++
++
A 58-year-old man is seen in his primary care physician’s office for evaluation of bilateral breast enlargement. This has been present for several months and is accompanied by mild pain in both breasts. He reports no other symptoms. His other medical conditions include coronary artery disease with a history of congestive heart failure, atrial fibrillation, obesity, and type 2 diabetes mellitus. His current medications include lisinopril, spironolactone, furosemide, insulin, and digoxin. He denies illicit drug use and has fathered three children. Examination confirms bilateral breast enlargement with palpable glandular tissue that measures 2 cm bilaterally. Which of the following statements regarding his gynecomastia is true?
++
++
++
A. He should be referred for mammography to rule out breast cancer.
++
++
B. His gynecomastia is most likely due to obesity, with adipose tissue present in the breast.
++
++
C. Serum testosterone, LH, and FSH should be measured to evaluate for androgen insensitivity.
++
++
D. Spironolactone should be discontinued, and he should be followed for regression.
++
++
E. Liver function testing should be performed to screen for cirrhosis.
+
++
The answer is D. (Chap. 411) Gynecomastia is a relatively common complaint in men and may be caused by either obesity with adipose tissue expansion in the breast or by an increased estrogen-to-androgen ratio in which there is true glandular enlargement, as in this case. If the breast is unilaterally enlarged or if it is hard or fixed to underlying tissue, mammography is indicated. Alternatively, if cirrhosis or a causative drug is present, these may be adequate explanations, particularly when gynecomastia develops later in life in previously fertile men. If the breast tissue is >4 cm or there is evidence of very small testes and no causative drugs or liver disease, a search for alterations in serum testosterone, LH, FSH, estradiol, and human chorionic gonadotropin (hCG levels) should be undertaken. An androgen deficiency or resistance syndrome may be present, or an hCG-secreting tumor may be found. In this case, spironolactone is the likely culprit, and it may be stopped or switched to eplerenone and gynecomastia reassessed.
++
++
++
All the following drugs may interfere with testicular function EXCEPT:
++
++
++
++
++
++
++
++
++
++
++
+
++
The answer is C. (Chap. 411) Many drugs may interfere with testicular function through a variety of mechanisms. Cyclophosphamide damages the seminiferous tubules in a dose- and time-dependent fashion and causes azoospermia within a few weeks of initiation. This effect is reversible in approximately half of these patients. Ketoconazole inhibits testosterone synthesis. Spironolactone causes a blockade of androgen action which may also cause gynecomastia. Glucocorticoids lead to hypogonadism predominantly through inhibition of hypothalamic-pituitary function. Sexual dysfunction has been described as a side effect of therapy with β-blockers. However, there is no evidence of an effect on testicular function. Most reports of sexual dysfunction were in patients receiving older β-blockers such as propranolol and timolol.
++
++
++
Clinical signs and findings of the presence of ovulation include all of the following EXCEPT:
++
++
++
A. Detection of urinary LH surge
++
++
B. Estrogen peak during secretory phase of menstrual cycle
++
++
C. Increase in basal body temperature >0.5°F in second half of menstrual cycle
++
++
D. Presence of mittelschmerz
++
++
E. Progesterone level >5 ng/mL 7 days before expected menses
+
++
The answer is B. (Chap. 412) Women who have regular monthly bleeding cycles that do not vary by >4 days generally have ovulatory cycles, but several other indicators suggest that ovulation is likely. These include the presence of mittelschmerz, which is described as midcycle pelvic discomfort that is thought to be caused by rapid expansion of the dominant follicle at the time of ovulation or premenstrual symptoms such as breast tenderness, bloating, and food cravings. Additional objective parameters suggest the presence of ovulation including a progesterone level >5 ng/mL 7 days before expected menses, an increase in basal body temperature >0.5°F in the second half of the menstrual cycle, and detection of urinary LH surge. Estrogen levels are elevated at the time of ovulation and during the secretory phase of the menstrual cycle but are not useful in detection of ovulation.
++
++
++
In the developmental progression from childhood through puberty to menopause, all of the following statements regarding levels of FSH and LH are true EXCEPT:
++
++
++
A. FSH is suppressed from birth to 20 months of age.
++
++
B. LH is increased during the neonatal year (birth to 20 months).
++
++
C. LH and FSH levels are reduced during childhood before puberty.
++
++
D. At the onset of puberty, pulsatile gonadotropin-releasing hormone (GnRH) drives pituitary FSH and LH levels.
++
++
E. LH and FSH levels rise sharply after menopause.
+
++
The answer is A. (Chap. 412) After birth and the loss of placenta-derived steroids, gonadotropin levels rise. FSH levels are much higher in girls than in boys. This rise in FSH results in ovarian activation (evident on ultrasound) and increased inhibin B and estradiol levels. Studies that have identified mutations in TAC3, which encodes neurokinin B, and its receptor, TAC3R, in patients with GnRH deficiency indicate that both are involved in control of GnRH secretion and may be particularly important at this early stage of development. By 12–20 months of age, the reproductive axis is again suppressed, and a period of relative quiescence persists until puberty. At the onset of puberty, pulsatile GnRH secretion induces pituitary gonadotropin production. In the early stages of puberty, LH and FSH secretion are apparent only during sleep, but as puberty develops, pulsatile gonadotropin secretion occurs throughout the day and night. Gonadotropin levels are cyclic during the reproductive years and increase dramatically with the loss of negative feedback that accompanies menopause (Figure X-42).
+
++
++
++
++
++
Which of the following occurs first in the majority of girls with normal pubertal development?
++
++
++
A. Achieving peak height velocity
++
++
++
++
++
++
D. Development of pubic hair
++
++
E. Development of axillary hair.
+
++
The answer is C. (Chap. 412) The first menstrual period (menarche) occurs relatively late in the series of developmental milestones that characterize normal pubertal development. Menarche is preceded by the appearance of pubic and then axillary hair (adrenarche) as a result of maturation of the zona reticularis in the adrenal gland and increased adrenal androgen secretion, particularly dehydroepiandrosterone (DHEA). The triggers for adrenarche remain unknown but may involve increases in body mass index, as well as in utero and neonatal factors. Menarche is also preceded by breast development (thelarche). The breast is exquisitely sensitive to the very low levels of estrogen that result from peripheral conversion of adrenal androgens and the low levels of estrogen secreted from the ovary early in pubertal maturation. Breast development precedes the appearance of pubic and axillary hair in approximately 60% of girls. The interval between the onset of breast development and menarche is approximately 2 years. There has been a gradual decline in the age of menarche over the past century, attributed in large part to improvement in nutrition, and there is a relationship between adiposity and earlier sexual maturation in girls.
++
++
++
The Women’s Health Initiative study investigated hormonal therapy in postmenopausal women. The study was stopped early due to increased risk of which of the following diseases in the estrogen-only arm?
++
++
++
A. Deep venous thrombosis
++
++
++
++
++
++
++
++
+
++
The answer is E. (Chap. 413) The Women’s Health Initiative was the largest study of hormone therapy to date, including 27,000 postmenopausal women age 50 to 79 years who were followed for an average of 5–7 years. It was presumed that hormone replacement in this group of women would decrease cardiovascular risk. However, the trial was stopped early because of an unfavorable risk-benefit ratio in the estrogen-progestin arm and an increased risk of stroke that was not offset by lower coronary heart disease in the estrogen-only arm. Endometrial cancer risk was higher in patients with an intact uterus who were taking estrogen only. Use of progesterone eliminated this risk. Unopposed estrogen was associated with an increased risk of stroke that far outweighed the decreased risk of coronary heart disease. Estrogen-progestin together was associated with an increased risk of coronary heart disease. Osteoporosis risk was decreased in both the estrogen and estrogen-progestin groups. Venous thromboembolism risk was higher in both treatment groups as well. These therapies do reduce important menopausal symptoms such as hot flashes and vaginal drying. This seminal study caused a dramatic reevaluation of the use of estrogen and progesterone in postmenopausal women to reduce cardiovascular risk. In addition, it reiterated the importance of well-designed clinical studies to test accepted dogma.
++
++
++
All of the following are traditional contraindications for oral hormone replacement therapy in postmenopausal women EXCEPT:
++
++
++
++
++
B. Blood clotting disorder
++
++
++
++
D. Coronary heart disease risk over ensuing 10 years of 5%–10%
++
++
E. Unexplained vaginal bleeding
+
++
The answer is D. (Chap. 413) Traditional contraindications for oral hormone replacement therapy are unexplained vaginal bleeding; active liver disease; history of venous thromboembolism due to pregnancy, oral contraceptive use, or an unknown etiology; blood clotting disorder; history of breast or endometrial cancer; and diabetes. Ten-year risk of coronary heart disease, based on the Framingham Coronary Heart Disease Risk Score, indicating a risk of 5%–10% is not a traditional contraindication for oral hormone replacement therapy.
++
++
++
A couple that has been married for 5 years have been attempting to conceive a child for the last 12 months. Despite regular intercourse, they have not achieved pregnancy. They are both 32 years of age and have no medical problems. Neither partner is taking medications. Which of the following is the most common cause of infertility?
++
++
++
++
++
++
++
++
++
++
++
+
++
The answer is C. (Chap. 414) Infertility, defined as the inability to conceive after 12 months of unprotected intercourse, is a common problem in the United States, with an estimated 15% of couples affected. Initial evaluation should include an evaluation of current menstrual history, counseling regarding the appropriate timing of intercourse, and education regarding modifiable risk factors such as drug use, alcohol intake, smoking, caffeine, and obesity. Male factors are at the root of approximately 25% of cases of infertility, unexplained infertility is found in 17% of cases, and female causes underlie 58% of infertility cases. Among the female causes, the most common is amenorrhea/ovulatory dysfunction, which is present in 46% of cases. This is most frequently due to hypothalamic or pituitary causes or polycystic ovary syndrome. Tubal defects and endometriosis are less common.
++
++
++
A couple seeks advice regarding infertility. The female partner is 35 years old. She has never been pregnant and was taking oral contraceptive pills from age 20 until age 34. It is now 16 months since she discontinued her oral contraceptives. She is having menstrual cycles approximately once every 35 days, but occasionally will go as long as 60 days between cycles. Most months, she develops breast tenderness about 2–3 weeks after the start of her menstrual cycle. When she was in college, she was treated for Neisseria gonorrhoeae that was diagnosed when she presented to the student health center with a fever and pelvic pain. She otherwise has no medical history. She works about 60 hours weekly as a corporate attorney and exercises daily. She drinks coffee daily and alcohol at social occasions only. Her body mass index (BMI) is 19.8 kg/m2. Her husband, who is 39 years old, accompanies her to the evaluation. He also has never had children. He was married previously from the ages of 24–28. He and his prior wife attempted to conceive for about 15 months but were unsuccessful. At that time, he was smoking marijuana on a daily basis and attributed their lack of success to his drug use. He has now been completely free of drugs for 9 years. He suffers from hypertension and is treated with lisinopril 10 mg daily. He is not obese (BMI 23.7 kg/m2). They request evaluation for their infertility and request help with conception. Which of the following statements is true regarding their infertility and likelihood of success in conception?
++
++
++
A. Determination of ovulation is not necessary in the female partner because most of her cycles occur regularly and she develops breast tenderness midcycle, indicating ovulation.
++
++
B. Lisinopril should be discontinued immediately because of the risk of birth defects associated with its use.
++
++
C. The female partner should be assessed for tubal patency by a hysterosalpingogram. If significant scarring is found, in vitro fertilization should be strongly considered to decrease the risk of ectopic pregnancy.
++
++
D. The prolonged use of oral contraceptives for >10 years has increased the risk of anovulation and infertility.
++
++
E. The use of marijuana by the male partner is directly toxic to sperm motility, and this is the likely cause of their infertility.
+
++
The answer is C. (Chap. 414) Evaluation of infertility should include evaluation of common male and female factors that could be contributing. Abnormalities of menstrual function are the most common cause of female infertility, and initial evaluation of infertility should include evaluation of ovulation and assessment of tubal and uterine patency. The female partner reports an episode of gonococcal infection with symptoms of pelvic inflammatory disease, which would increase her risk of infertility due to tubal scarring and occlusion. A hysterosalpingogram is indicated. If there is evidence of tubal abnormalities, many experts recommend in vitro fertilization for conception because these women are at increased risk of ectopic pregnancy if conception occurs. The female partner reports some irregularity of her menses, suggesting anovulatory cycles, and thus, evidence of ovulation should be determined by assessing hormonal levels. There is no evidence that prolonged use of oral contraceptives affects fertility adversely (Farrow A, et al: Hum Reprod 17:2754, 2002). Angiotensin-converting enzyme inhibitors, including lisinopril, are known teratogens when taken by women but have no effect on chromosomal abnormalities in men. Recent marijuana use may be associated with increased risk of infertility, and in vitro studies of human sperm exposed to a cannabinoid derivative showed decreased motility (Whan LB, et al: Fertil Steril 85:653, 2006). However, no studies have shown long-term decreased fertility in men who previously used marijuana.
++
++
++
Which of the following forms of contraception has a theoretical efficacy of >90%?
++
++
++
++
++
++
++
++
++
++
++
+
++
The answer is E. (Chap. 414) All of the choices have a theoretical efficacy in preventing pregnancy of >90%. However, the actual effectiveness can vary widely. Spermicides have the greatest failure rate (21%). Barrier methods (e.g., condoms, cervical cap, diaphragm) have an actual efficacy between 82% and 88%. Oral contraceptives and intrauterine devices perform similarly, with 97% efficacy in preventing pregnancy in clinical practice.
++
++
++
A 30-year-old man, the father of three children, has had progressive breast enlargement during the last 6 months. He does not use any drugs. Laboratory evaluation reveals that both LH and testosterone are low. Further evaluation of this patient should include which of the following?
++
++
++
A. 24-Hour urine collection for the measurement of 17-ketosteroids
++
++
B. Blood sampling for serum glutamic oxaloacetic transaminase (SGOT) and serum alkaline phosphatase and bilirubin levels
++
++
++
++
D. Karyotype analysis to exclude Klinefelter syndrome
++
++
E. Measurement of estradiol and human chorionic gonadotropin (hCG) levels
+
++
The answer is E. (Chap. 414) Pathologic gynecomastia develops when the effective ratio of testosterone to estrogen is decreased due to diminished testosterone production (as in primary testicular failure) or increased estrogen production. The latter may arise from direct estradiol secretion by a testis stimulated by LH or hCG or from an increase in peripheral aromatization of precursor steroids, most notably androstenedione. Elevated androstenedione levels may result from increased secretion by an adrenal tumor (leading to an elevated level of urinary 17-ketosteroids) or decreased hepatic clearance in patients with chronic liver disease. A variety of drugs, including diethylstilbestrol, heroin, digitalis, spironolactone, cimetidine, isoniazid, and tricyclic antidepressants, also can cause gynecomastia. In this patient, the history of paternity and the otherwise normal physical examination indicate that a karyotype is unnecessary, and the bilateral breast enlargement essentially excludes the presence of carcinoma and thus the need for biopsy. The presence of low LH and testosterone levels suggests either estrogen or hCG production. Because of the normal testicular examination, a primary testicular tumor is not suspected. Carcinoma of the lung and germ cell tumors both can produce hCG, causing gynecomastia.
++
++
++
You are seeing a 36-year-old woman in clinic as her family practitioner. In her history, she reports no illness and is taking no medications. She does mention that she and her husband have been trying to conceive a child for the past 7 months but have been unsuccessful. Which of the following would be an appropriate response?
++
++
++
A. “You have likely entered menopause and cannot have a child.”
++
++
B. “We do not recommend evaluation by a fertility specialist until you and her husband have tried for at least 12 months.”
++
++
C. “I will refer you to an expert in fertility issues.”
++
++
D. “Most causes of infertility are related to the male. I suggest you have him evaluated.”
++
++
E. “Advancing age does not reduce a woman’s chance of becoming pregnant until she reaches menopause.”
+
++
The answer is C. (Chap. 414) The spectrum of infertility ranges from reduced conception rates or the need for medical intervention to irreversible causes of infertility. Infertility can be attributed primarily to male factors in 25% of couples and female factors in 58% of couples and is unexplained in about 17% of couples. Not uncommonly, both male and female factors contribute to infertility. Decreases in the ability to conceive as a function of age in women have led to recommendations that women >34 years old who are not at increased risk of infertility seek attention after 6 months, rather than 12 months as suggested for younger women, and receive an expedited workup and approach to treatment.
++
++
++
Which of the following ethnic populations in the United States has the highest risk of diabetes mellitus?
++
++
++
++
++
++
++
++
++
++
++
+
++
The answer is D. (Chap. 417) The risk of both type 1 and type 2 diabetes mellitus is rising in all populations, but the risk of type 2 diabetes is rising at a substantially faster rate. In the United States, the age-adjusted prevalence of diabetes mellitus is 7.1% in non-Hispanic whites, 7.5% in Asian Americans, 11.8% in Hispanics, and 12.6% in non-Hispanic blacks. Comparable data are not available for individuals belonging to American Indian, Alaska Native, or Pacific Islander populations, but the prevalence is thought to be even higher than in the non-Hispanic black population.
++
++
++
Which of the following defines normal glucose tolerance?
++
++
++
A. Fasting plasma glucose <100 mg/dL
++
++
B. Fasting plasma glucose <126 mg/dL following an oral glucose challenge
++
++
C. Fasting plasma glucose <100 mg/dL, plasma glucose <140 mg/dL following an oral glucose challenge, and hemoglobin A1C <5.6%
++
++
D. Hemoglobin A1C <5.6% and fasting plasma glucose <140 mg/dL
++
++
+
++
The answer is C. (Chap. 417) Glucose tolerance is classified into three categories: normal glucose tolerance, impaired glucose homeostasis, and diabetes mellitus. Normal glucose tolerance is defined by the following: fasting plasma glucose <100 mg/dL, plasma glucose <140 mg/dL following an oral glucose challenge, and hemoglobin A1C <5.6%. Abnormal glucose homeostasis is defined as a fasting plasma glucose of 100–125 mmol/dL or a plasma glucose of 140–199 following an oral glucose tolerance test or hemoglobin A1C of 5.7%–6.4%. Actual diabetes mellitus is defined by either a fasting plasma glucose >126 mg/dL, glucose of 200 mg/dL after an oral glucose tolerance test, or hemoglobin A1C ≥6.5%.
++
++
++
A 37-year-old obese woman presents to clinic for routine health evaluation. She reports that over the last year she has had two yeast infections treated with over-the-counter remedies and she frequently feels thirsty. She reports waking up at night to urinate. Which of the following studies is the most appropriate first test in evaluating the patient for diabetes mellitus?
++
++
++
++
++
B. Oral glucose tolerance test
++
++
C. Plasma C-peptide level
++
++
++
++
E. Random plasma glucose level
+
++
The answer is E. (Chap. 417) Because the patient has symptoms, she is not being screened for diabetes mellitus. For screening, the fasting plasma glucose or hemoglobin A1C is recommended. Because the patient has symptoms, a random plasma glucose of >200 mg/dL is adequate to diagnose diabetes mellitus. Other criteria include fasting plasma glucose >126 mg/dL, hemoglobin A1C >6.4%, or 2-hour plasma glucose >200 mg/dL during an oral glucose tolerance test. C-peptide is a useful tool to determine if the normal cleavage of insulin from its precursor is occurring. A normal C-peptide level with hypoglycemia suggests surreptitious insulin use, and a low C-peptide level with hyperglycemia suggests pancreatic failure.
++
++
++
A 27-year-old woman with mild obesity is seen by her primary care physician for increased thirst and polyuria. Diabetes mellitus is suspected, and a random plasma glucose of 211 mg/d confirms this diagnosis. Which of the following tests will strongly indicate that she has type 1 diabetes mellitus?
++
++
++
++
++
B. Peroxisome proliferator-activated receptor γ-2 polymorphism testing
++
++
++
++
D. Testing for human leukocyte antigen (HLA) DR3
++
++
E. There is no laboratory test to detect type 1 diabetes mellitus.
+
++
The answer is A. (Chap. 417) Type 1 diabetes mellitus often has a more severe presentation with diabetic ketoacidosis and often presents in younger individuals compared with type 2 diabetes; however, there are some cases where the distinction of type 1 from type 2 diabetes is not straightforward. Human leukocyte antigen (HLA) DR3 localization preferences exist for type 1 diabetes; several haplotypes are present in 40% of children with type 1 diabetes mellitus, but this is still the minority. Immunologic destruction of the β cell is the primary cause of disease in type 1 diabetes, and islet cell antibodies are commonly present. GAD, insulin, IA/ICA-512, and ZnT-8 are the most common targets. Commercially available assays for GAD-65 autoantibodies are widely available and can demonstrate antibodies in >85% of individuals with recent-onset type 1 diabetes. These autoantibodies are infrequently present in type 2 diabetes mellitus (5%–10%). There may be some residual insulin in the plasma in early type 1 diabetes; thus, this will not distinguish the two conditions reliably. Polymorphisms of the peroxisome proliferator-activated receptor γ-2 have been described in type 2 diabetes mellitus but cannot distinguish the two conditions.
++
++
++
You have admitted an 18-year-old patient to the adult medical intensive care unit for diabetic ketoacidosis (DKA). The patient was not known previously to be diabetic, but her mother notes that she had been “going to the bathroom a lot” recently and that “she had been really thirsty.” The patient’s BMI is 44 kg/m2. There is no family history of diabetes. You successfully treat the patient for her DKA and note that serum anti-GAD antibodies and anti–islet cell antibodies (ICA) sent on admission are not detected. The patient and her mother want to know what “type” of diabetes she has. You should tell them which of the following?
++
++
++
A. “Due to the young age of onset, you likely have type 1 diabetes.”
++
++
B. “Due to your presentation with diabetic ketoacidosis, you likely have type 1 diabetes.”
++
++
C. “I suspect you have maturity-onset diabetes of the young.”
++
++
D. “You likely have type 2 diabetes mellitus.”
++
++
E. “I suspect your diabetes was triggered by a virus.”
+
++
The answer is D. (Chap. 417) Individuals with type 2 diabetes mellitus (T2DM) often exhibit the following features: (1) develop diabetes after the age of 30 years; (2) are usually obese (80% are obese, but elderly individuals may be lean); (3) may not require insulin therapy initially; and (4) may have associated conditions such as insulin resistance, hypertension, cardiovascular disease, dyslipidemia, or polycystic ovary syndrome. In T2DM, insulin resistance is often associated with abdominal obesity (as opposed to hip and thigh obesity) and hypertriglyceridemia. Although most individuals diagnosed with T2DM are older, the age of diagnosis is declining, and there is a marked increase among overweight children and adolescents. The age of the patient should not be the sole basis for determining the type of diabetes present. Some individuals with phenotypic T2DM present with diabetic ketoacidosis but lack autoimmune markers and may be later treated with oral glucose-lowering agents rather than insulin (this clinical picture is sometimes referred to as ketosis-prone T2DM). Monogenic forms of diabetes (maturity-onset diabetes of the young) should be considered in those with diabetes onset at <30 years of age, an autosomal pattern of diabetes inheritance (which this patient lacks), and the lack of nearly complete insulin deficiency.
++
++
++
A patient is evaluated in the emergency department for complications of diabetes mellitus due to an episode of life stressors. All of the following laboratory tests are consistent with the diagnosis of DKA EXCEPT:
++
++
++
++
++
++
++
C. Markedly positive plasma ketones
++
++
D. Normal serum potassium
++
++
E. Plasma osmolality of 380 mOsm/mL
+
++
The answer is E. (Chap. 418) Diabetic ketoacidosis and hyperglycemic hyperosmolar state exist on a spectrum with diabetic ketoacidosis more common in patients with type 1 diabetes mellitus, but does occur with some frequency in patients with T2DM. Both conditions include hyperglycemia, dehydration, absolute or relative insulin deficiency, and acid-base abnormalities. Ketosis is more common in diabetic ketoacidosis. In diabetic ketoacidosis, glucose normally ranges from 250–600 mg/dL, whereas it is frequently 600–1200 mg/dL in hyperglycemic hyperosmolar state. Sodium is often mildly depressed in ketoacidosis and is preserved in hyperosmolar state. Potassium is normal to elevated in diabetic ketoacidosis and normal in hyperglycemic hyperosmolar patients. Magnesium, chloride, and phosphate are normal in both conditions. Creatinine may be slightly elevated in diabetic ketoacidosis but is often moderately elevated in hyperglycemic hyperosmolar state. Plasma ketones may be slightly positive in hyperosmolar patients, but are always strongly positive in diabetic ketoacidosis. Because hyperosmolarity is the hallmark of hyperglycemic hyperosmolar patients, they have an osmolarity of 330–380 mOsm/mL, whereas patients with diabetic ketoacidosis typically have a slightly elevated plasma osmolarity ranging from 300–320 mOsm/mL. Serum bicarbonate is markedly depressed in diabetic ketoacidosis and normal or slightly depressed in hyperosmolar state. Arterial pH is depressed at <7.3 in ketoacidosis and >7.3 in hyperosmolar state. Finally, the anion gap is wide in diabetic ketoacidosis and normal to slightly elevated in hyperglycemic hyperosmolar state.
++
++
++
Pick the correct combination of onset of action and duration of action for the following insulins.
++
++
++
++
++
++
++
++
++
++
++
++
+
++
The answer is D. (Chap. 418) Insulin preparations can be divided into short-acting and long-acting insulins. The short-acting insulins include regular and new preparations including aspart, glulisine, and lispro. Regular insulin has an onset of action of 0.5–1 hour and is effective for 4–6 hours. The other three short-acting insulins have an onset of action of <0.25 hour and are effective for 3–4 hours. Long-acting insulins include detemir, glargine, and NPH. Detemir and glargine have an onset of action of 1–4 hours and last up to 24 hours, whereas NPH has an onset of action of 1–4 hours and is effective for 10–16 hours. These insulins have a number of combination preparations that take advantage of the different durations of onset and action to provide optimal efficacy and compliance.
++
++
++
A 54-year-old woman is diagnosed with type 2 diabetes mellitus after a routine follow-up for impaired fasting glucose showed that her hemoglobin A1C is now 7.6%. She has attempted to lose weight and exercise with no improvement in her hemoglobin A1C, and drug therapy is now recommended. She has mild systemic hypertension that is well controlled and no other medical conditions. Which of the following is the most appropriate first-line therapy?
++
++
++
++
++
++
++
++
++
++
++
+
++
The answer is D. (Chap. 418) First-line oral therapy for patients with T2DM is metformin. It is contraindicated in patients with a glomerular filtration rate <60 mL/min, any form of acidosis, congestive heart failure, liver disease, or severe hypoxemia, but is well tolerated in most individuals. Insulin secretagogues, biguanides, α-glucosidase inhibitors, thiazolidinediones, GLP-1 agonists, dipeptidyl peptidase-4 (DPP-IV) inhibitors, and insulin have all been approved as monotherapy for T2DM. Because of extensive clinical experience, favorable side effect profile, and relatively low cost, metformin is the recommended first-line agent. It has additional benefits of promotion of mild weight loss, lower insulin levels, and mild improvements in lipid profile. Sulfonylureas such as glyburide, GLP-1 agonists such as exenatide, and insulin DPP-IV inhibitors such as sitagliptin may be appropriate as combination therapy but are not considered first-line therapy for most patients.
++
++
++
A 21-year-old woman with a history of type 1 diabetes mellitus is brought to the emergency department with nausea, vomiting, lethargy, and dehydration. Her mother notes that she stopped taking insulin 1 day before presentation. She is lethargic, has dry mucous membranes, and is obtunded. Blood pressure is 80/40 mmHg, and heart rate is 112 bpm. Heart sounds are normal. Lungs are clear. The abdomen is soft, and there is no organomegaly. She is responsive and oriented × 3 but diffusely weak. Serum sodium is 126 mEq/L, potassium is 4.3 mEq/L, magnesium is 1.2 mEq/L, blood urea nitrogen is 76 mg/dL, creatinine is 2.2 mg/dL, bicarbonate is 10 mEq/L, and chloride is 88 mEq/L. Serum glucose is 720 mg/dL. All the following are appropriate management steps EXCEPT:
++
++
++
++
++
++
++
++
++
++
++
+
++
The answer is A. (Chap. 418) Diabetic ketoacidosis is an acute complication of diabetes mellitus. It results from a relative or absolute deficiency of insulin combined with a counterregulatory hormone excess. In particular, a decrease in the ratio of insulin to glucagons promotes gluconeogenesis, glycogenolysis, and the formation of ketone bodies in the liver. Ketosis results from an increase in the release of free fatty acids from adipocytes, with a resultant shift toward ketone body synthesis in the liver. This is mediated by the relationship between insulin and the enzyme carnitine palmitoyltransferase I. At physiologic pH, ketone bodies exist as ketoacids, which are neutralized by bicarbonate. As bicarbonate stores are depleted, acidosis develops. Clinically, these patients have nausea, vomiting, and abdominal pain. They are dehydrated and may be hypotensive. Lethargy and severe central nervous system depression may occur. The treatment centers on replacement of the body’s insulin, which will result in cessation of the formation of ketoacids and improvement of the acidotic state. Assessment of the level of acidosis may be done with an arterial blood gas. These patients have an anion gap acidosis and often a concomitant metabolic alkalosis resulting from volume depletion. Volume resuscitation with intravenous fluids is critical. Many electrolyte abnormalities may occur. Total-body sodium, potassium, and magnesium are depleted in these patients. As a result of the acidosis, intracellular potassium may shift out of cells and cause a normal or even elevated potassium level. However, with improvement in the acidosis, the serum potassium rapidly falls. Therefore, potassium repletion is critical despite the presence of a “normal” level. Because of the osmolar effects of glucose, fluid is drawn into the intravascular space. This results in a drop in the measured serum sodium. There is a drop of 1.6 mEq/L in serum sodium for each rise of 100 mg/dL in serum glucose. In this case, the serum sodium will improve with hydration alone. The use of 3% saline is not indicated because the patient has no neurologic deficits, and the expectation is for rapid resolution with IV fluids alone.
++
++
++
You are seeing a 28-year-old woman with longstanding type 1 diabetes on insulin. She tells you that she and her husband have decided to try to conceive. Which of the following is true regarding reproductive issues and diabetes?
++
++
++
A. Women with diabetes have a reduced reproductive capacity.
++
++
B. Insulin crosses the placenta and may affect the fetus adversely.
++
++
C. The patient should expect her insulin requirements to increase during pregnancy.
++
++
D. High maternal serum glucose increases the risk of fetal abnormalities.
++
++
E. The most crucial period of glycemic control is in the third trimester to avoid fetal malformations.
+
++
The answer is D. (Chap. 418) Reproductive capacity in either men or women with diabetes mellitus appears to be normal. Menstrual cycles may be associated with alterations in glycemic control in women with diabetes. Pregnancy is associated with marked insulin resistance; the increased insulin requirements often precipitate diabetes and lead to the diagnosis of gestational diabetes mellitus. Glucose, which at high levels is a teratogen to the developing fetus, readily crosses the placenta, but insulin does not. Thus, hyperglycemia from the maternal circulation may stimulate insulin secretion in the fetus. The anabolic and growth effects of insulin may result in macrosomia. Pregnancy in individuals with known diabetes requires meticulous planning and adherence to strict treatment regimens. Intensive diabetes management and normalization of the hemoglobin A1C are essential for individuals with existing diabetes who are planning pregnancy. The most crucial period of glycemic control is soon after fertilization. The risk of fetal malformations is increased 4–10 times in individuals with uncontrolled diabetes at the time of conception, and normal plasma glucose during the preconception period and throughout the periods of organ development in the fetus should be the goal.
++
++
++
Which of the following regarding care of the hospitalized diabetic patient is true?
++
++
++
A. General anesthesia leads to insulin sensitization and higher risk for hypoglycemia.
++
++
B. A greater degree of hyperglycemia during hospitalization has not been associated with worse infectious outcomes.
++
++
C. In clinical trials, strict glycemic control (goal, 81–108 mg/dL) is superior to moderate glycemic control (goal, 140 mg/dL).
++
++
D. The initiation of total parenteral nutrition is associated with increased insulin requirements.
++
++
E. In critically ill patients, subcutaneous insulin is invariably preferred over IV insulin.
+
++
The answer is D. (Chap. 418) Virtually all medical and surgical subspecialties are involved in the care of hospitalized patients with diabetes. Hyperglycemia, whether in a patient with known diabetes or in someone without known diabetes, appears to be a predictor of poor outcome in hospitalized patients. General anesthesia, surgery, infection, or concurrent illness raises the levels of counterregulatory hormones (cortisol, growth hormone, catecholamines, and glucagon) and cytokines that may lead to transient insulin resistance and hyperglycemia. In a number of cross-sectional studies of patients with diabetes, a greater degree of hyperglycemia was associated with worse cardiac, neurologic, and infectious outcomes. In some studies, patients who do not have preexisting diabetes but who develop modest blood glucose elevations during their hospitalization appear to benefit from achieving near-normoglycemia using insulin treatment. However, a large randomized clinical trial (Normoglycemia in Intensive Care Evaluation Survival Using Glucose Algorithm Regulation [NICESUGAR]) of individuals in the intensive care unit (ICU; most of whom were receiving mechanical ventilation) found an increased mortality rate and a greater number of episodes of severe hypoglycemia with very strict glycemic control (target blood glucose of 4.5–6 mmol/L or 81–108 mg/dL) compared to individuals with a more moderate glycemic goal (mean blood glucose of 8 mmol/L or 144 mg/dL). Total parenteral nutrition (TPN) greatly increases insulin requirements. In addition, individuals not previously known to have diabetes may become hyperglycemic during TPN and require insulin treatment. Insulin infusions are preferred in the ICU or in a clinically unstable setting. The absorption of subcutaneous insulin may be variable in such situations. Insulin infusions can also effectively control plasma glucose in the perioperative period and when the patient is unable to take anything by mouth.
++
++
++
In Figure X-62, what is the primary finding seen in this patient’s fundus?
+
++
++
++
++
++
++
++
++
++
++
++
+
++
The answer is C. (Chap. 419) Diabetic retinopathy is the leading cause of blindness in individuals between the ages of 20 and 74 in the United States. Individuals with diabetes are 25 times more likely to go legally blind than individuals without diabetes. Diabetic retinopathy is classified into two stages: nonproliferative and proliferative. Nonproliferative retinopathy typically appears late in the first decade or early in the second decade of the disease. Characteristic findings include cotton wool spots, blot hemorrhages, and retinal vascular microaneurysms. Mild proliferative retinopathy may progress to more extensive disease, characterized by changes in venous vessel caliber, intraretinal microvascular abnormalities, and more numerous microaneurysms and hemorrhages. Pathophysiologically, there is loss of retinal pericytes, increased retinal vascular permeability, alterations in retinal blood flow, and abnormal retinal microvasculature. Severe nonproliferative diabetic retinopathy creates retinal hypoxemia and establishes the environment for development of proliferative retinopathy. Neovascularization, as shown in this patient’s fundus, is the hallmark of proliferative retinopathy. Neovascular vessels appear at the optic disc. The most effective therapy for the treatment of diabetic retinopathy is prevention with intensive glycemic and blood pressure control. However, in established diabetic retinopathy, improved glycemic control leads to transient worsening of the disease. When proliferative retinopathy and neovascularization are present, retinal laser photocoagulation is required.
++
++
++
Which of the following patients should be treated with either an angiotensin-converting enzyme (ACE) inhibitor or angiotensin receptor blocker?
++
++
++
A. A 24-year-old woman with type 1 diabetes with two positive spot microalbuminuria tests 1 week apart
++
++
B. A 32-year-old woman with type 1 diabetes with a blood glucose of 328 mg/dL and a positive spot microalbuminuria test
++
++
C. A 48-year-old man with type 2 diabetes with a positive spot microalbuminuria test 1 week after starting a new exercise program
++
++
D. A 56-year-old man with type 2 diabetes with two positive spot microalbuminuria tests 3 months apart
++
++
E. A 62-year-old man with type 2 diabetes and hypertension with a positive spot microalbuminuria test and a blood pressure on day of testing of 190/118 mmHg
+
++
The answer is D. (Chap. 419) Diabetes mellitus is the most common cause of chronic kidney disease, end-stage renal disease, and chronic kidney disease requiring renal replacement therapy. In the first 5 years after diabetes onset, glomerular hyperfiltration and increased glomerular filtration rate are seen. The glomerular basement membrane subsequently thickens, with concomitant mesangial volume and glomerular hypertrophy. Typically within 5–10 years, many individuals will begin to excrete small amounts of albumin in their urine. It is recommended to screen for albumin excretion annually with a 24-hour collection or spot albumin-to-creatinine ratio. Microalbuminuria is defined as >30–299 g/d in a 24-hour collection or 30–299 g/mg creatinine in a spot collection. However, interpretation may be clouded by conditions known to transiently increase albumin excretion including urinary tract infection, hematuria, heart failure, febrile illness, severe hyperglycemia (option B), severe hypertension (option E), pregnancy, and vigorous exercise (option C). If testing is positive, it should be repeated within 3–6 months, and treatment should begin at that time. This makes option A too soon for repeat testing and option D the most appropriate answer to the question. Although direct comparisons have not been done, experts believe that angiotensin-converting enzyme (ACE) inhibitors and angiotensin receptor blockers are equivalent in the treatment of albuminuria and diabetic nephropathy.
++
++
++
A 58-year-old woman with type 2 diabetes mellitus is evaluated by her primary care provider for a tingling sensation in her hands and feet. She has had type 2 diabetes for 15 years with intermittently poor control. Her most recent hemoglobin A1C was 7.9%. She is currently managed with insulin detemir 40 units daily and metformin 1000 mg daily. On neurologic examination, there is loss of deep tendon reflexes at the ankles bilaterally. Deep tendon reflexes are 2+ at the knees, biceps, and triceps. Sensation is decreased to pinprick and light touch bilaterally to the ankle and wrists. She also has difficulty ascertaining if the great toe is being held in the up or down position when her eyes are closed. She finds it difficult to sleep at night sometimes due to the pain in her legs. She is diagnosed with distal sensory polyneuropathy due to her diabetes. Which of the following medications has been approved by the U.S. Food and Drug Administration for the treatment of pain associated with diabetic neuropathy?
++
++
++
++
++
++
++
++
++
++
++
+
++
The answer is D. (Chap. 419) Diabetic neuropathy occurs in as many as 50% of individuals with longstanding type 1 and type 2 diabetes and can manifests as polyneuropathy, mononeuropathy, and/or autonomic neuropathy. Like other complications of diabetes, the likelihood of developing neuropathy depends on the duration of the disease and the degree of glycemic control. The most common form of neuropathy is distal symmetric polyneuropathy, which typically presents as distal sensory loss and pain. However, as many as 50% of individuals will have no symptoms at all. Alternatively, hyperesthesia, paresthesia, and dysesthesia may also occur. Clinically, patients often complain of a pins-and-needles sensation in the distal extremities. Other symptoms include burning, numbness, or a sharp pain. Both the hands and feet may be affected, but usually, it starts in the lower extremities. Pain is worse at rest and at night. An acute form of diabetic neuropathy may worsen in the setting of improved glycemic control. As the neuropathy progresses, the pain may subside with worsening numbness and sensory deficit. Physical examination shows sensory loss to light touch and pinprick with loss of ankle deep tendon reflexes and abnormal position sense. Treatment of diabetic neuropathy is difficult. Improved glycemic control may improve nerve conduction velocity, but symptoms of neuropathy may not improve. Other neurotoxins, such as alcohol, smoking, and vitamin deficiencies, that could worsen neuropathy should be avoided. Multiple agents have been used to attempt to treat painful neuropathy, but the results of treatment are less than ideal. Only two agents—duloxetine and pregabalin—have been approved by the U.S. Food and Drug Administration for the treatment of diabetic neuropathy. Other agents that are sometimes used off-label include amitriptyline, gabapentin, valproate, and opioids.
++
++
++
Plasma glucose is normally tightly regulated in the body, with fasting levels between 70 and 110 mg/dL. When the blood glucose falls below 80–85 mg/dL, which of the following physiologic changes is the first to occur?
++
++
++
A. Decrease in growth hormone
++
++
B. Decrease in insulin secretion
++
++
++
++
D. Increase in epinephrine
++
++
+
++
The answer is B. (Chap. 420) Maintenance of euglycemia involves a number of systems to lower elevated blood glucose, but also to restore normal levels when hypoglycemia is present or impending. This is particularly important for neurologic functioning because the brain cannot synthesize glucose and has only a few minutes’ supply stored as glycogen. In a nondiabetic individual, the normal fasting plasma glucose level is normally tightly controlled between 70 and 110 mg/dL. When the plasma glucose begins to fall below about 80–85 mg/dL, the first line of defense to protect against development of hypoglycemia is to decrease insulin secretion. When this occurs, hepatic glycogenolysis and gluconeogenesis increase. In addition, lowered insulin levels leads to decreased peripheral glucose utilization. If the glucose continues to fall to about 65–70 mg/dL, other protective mechanisms are employed. The second line of defense against hypoglycemia is glucagon secretion, which further stimulates hepatic gluconeogenesis. Epinephrine may also be secreted, although it is not normally critical unless glucagon is deficient. Cortisol and growth hormone are secreted later in the pathway when hypoglycemia is prolonged greater than 4 hours. These hormones have no role in acute hypoglycemia.
++
++
++
A 25-year-old healthcare worker is seen for evaluation of recurrent hypoglycemia. She has had several episodes at work over the past year in which she feels shaky, anxious, and sweaty; she measures her fingerstick glucose, and it is 40–55 mg/dL. This has been confirmed with a plasma glucose level during one episode of 50 mg/dL. She then drinks orange juice and feels better. These episodes have not happened outside the work environment. Aside from oral contraceptives, she takes no medications and is otherwise healthy. Which of the following tests is most likely to demonstrate the underlying cause of her hypoglycemia?
++
++
++
++
++
B. Measurement of fasting insulin and glucose levels
++
++
C. Measurement of fasting insulin, glucose, and C-peptide levels
++
++
D. Measurement of insulin, glucose, and C-peptide levels during an symptomatic episode
++
++
E. Measurement of plasma cortisol
+
++
The answer is D. (Chap. 420) The patient presents with recurrent episodes of hypoglycemia that meet the Whipple triad of symptoms: (1) symptoms of hypoglycemia; (2) low plasma glucose concentration measured with a precise method (not a glucose monitor); and (3) relief of symptoms with raising the plasma glucose level. The differential starts with measuring insulin levels during hypoglycemia. The levels must be obtained during an episode to be interpretable. If insulin is elevated, it suggests either endogenous hyperproduction from an insulin-secreting tumor or exogenous administration causing factitious hypoglycemia. Because C-peptide is cleaved from native proinsulin to make the secreted product, it will be high in the case of endogenous hyperinsulinemia and will be low during an episode of factitious hypoglycemia. Surreptitious ingestion of sulfonylurea could cause hypoglycemia along with high insulin and C-peptide levels since the drug stimulates pancreatic insulin secretion. In this case, a sulfonylurea drug screen would be indicated. Red flags in this case that point to surreptitious insulin use include the patient being a healthcare worker and the presence of symptoms only at work. Other groups in which this is common are relatives of patients with diabetes and patients with a history of other factitious disorders. It is possible that the patient has an insulin-secreting β cell tumor, but this is much less likely, and symptoms would be present during times other than work. Evaluation is aimed at demonstrating that pancreatic insulin secretion is suppressed during the episode of hypoglycemia. Although a failure of counterregulatory hormones can produce hypoglycemia, this is a rare cause of hypoglycemia, and evaluation should be aimed at this only after surreptitious use is ruled out.
++
++
++
All of the following statements regarding hypoglycemia in diabetes mellitus are true EXCEPT:
++
++
++
A. Individuals with type 2 diabetes mellitus experience less hypoglycemia than those with type 1 diabetes mellitus.
++
++
B. From 2%–4% of deaths in type 1 diabetes mellitus are directly attributable to hypoglycemia.
++
++
C. Recurrent episodes of hypoglycemia predispose to the development of autonomic failure with defective glucose counterregulation and hypoglycemia unawareness.
++
++
D. The average person with type 1 diabetes mellitus has two episodes of symptomatic hypoglycemia weekly.
++
++
E. Thiazolidinediones and metformin cause hypoglycemia more frequently than sulfonylureas.
+
++
The answer is E. (Chap. 420) The most common cause of hypoglycemia is related to the treatment of diabetes mellitus. Individuals with type 1 diabetes mellitus (T1DM) have more symptomatic hypoglycemia than individuals with T2DM. On average, those with T1DM experience two episodes of symptomatic hypoglycemia weekly; and at least once yearly, individuals with T1DM will have a severe episode of hypoglycemia that is at least temporarily disabling. It is estimated that 6%–10% of individuals with T1DM will die from hypoglycemia. In addition, recurrent episodes of hypoglycemia in T1DM contribute to the development of hypoglycemia-associated autonomic failure. Clinically, this is manifested as hypoglycemia unawareness and defective glucose counterregulation, with lack of glucagon and epinephrine secretion as glucose levels fall. Individuals with T2DM are less likely to develop hypoglycemia. However, once an individual with T2DM requires insulin, the likelihood of symptomatic hypoglycemia increases, and although the incidence of hypoglycemia is overall lower in T2DM, the absolute number of individuals with T2DM with hypoglycemia episodes is far greater than those with T1DM given the higher prevalence of T2DM. Medications that are associated with hypoglycemia in T2DM are insulin and insulin secretagogues, such as sulfonylureas. Metformin, thiazolidinediones, α-glucosidase inhibitors, glucagon-like peptide-1 receptor agonists, and DPP-IV inhibitors should not cause hypoglycemia. However, when these medications are combined with another class of medications known to cause hypoglycemia, they do increase the risk of hypoglycemic episodes.
++
++
++
An 18-year-old man presents with severe mid-abdominal pain radiating to his back. Physical examination reveals a temperature of 38.0°C, blood pressure of 95/55 mmHg, heart rate of 110 bpm, and respiratory rate of 18 breaths/min, with room air oxygen saturation 96%. His abdomen is diffusely tender with voluntary guarding and no rebound tenderness. There is enlargement of the liver and spleen. He also has eruptive xanthomas on his hands, feet, and legs. His lipase is 2300 U/L, and he has a fasting triglyceride level of 1019 mg/dL. After appropriate evaluation, he is presumed to have pancreatitis and lipoprotein lipase deficiency. He stabilizes without complication and is ready for discharge after 4 days. What do you recommend for treatment?
++
++
++
A. Dietary fat restriction 15 g/d
++
++
B. Fish oil supplementation
++
++
C. Gemfibrozil 600 mg twice a day (bid)
++
++
D. Nicotinic acid sustained-release 250 mg bid
++
++
E. Simvastatin 20 mg daily
+
++
The answer is A. (Chap. 421) Lipoprotein lipase (LPL) deficiency is a very rare autosomal recessive disorder that results in elevated fasting triglyceride levels because the absence of LPL results in the inability of chylomicrons to undergo hydrolysis of triglycerides. Thus, circulating levels of chylomicrons, very-low-density lipoprotein (VLDL), and triglycerides are high. Fasting triglyceride levels are typically >1000 mg/dL. LPL deficiency has an incidence of approximately 1 in 1,000,000 in the population. The disease typically presents in childhood or young adulthood with recurrent episodes of pancreatitis. Ophthalmologic examination may show lipemia retinalis with an opalescent appearance to the retinal blood vessels. Eruptive xanthomas are small, yellowish-white papules and may appear on the back, buttocks, and extensor surfaces of the arms and legs. Hepatosplenomegaly occurs because of uptake of the circulating chylomicrons by the reticuloendothelial system. Primary treatment of the disorder is to restrict dietary fat to 15 g/d or less. If dietary fat restriction alone is not sufficient to control the triglyceride level, fish oil has been useful in some patients.
++
++
++
A 32-year-old man is evaluated at a routine clinic visit for coronary risk factors. He reports no tobacco use, his systemic blood pressure is normal, and he does not have diabetes. He is otherwise healthy. His family history is notable for high cholesterol in his mother and maternal grandfather and grandmother. Physical examination shows tendon xanthomas. A fasting cholesterol is notable for a low-density lipoprotein (LDL) cholesterol level of 387 mg/dL. Which of the following is the most likely genetic disorder affecting this individual?
++
++
++
A. Apolipoprotein (apo) A-V deficiency
++
++
B. Familial defective apoB-100
++
++
C. Familial hepatic lipase deficiency
++
++
D. Familial hypercholesterolemia
++
++
E. Lipoprotein lipase deficiency
+
++
The answer is D. (Chap. 421) Familial hypercholesterolemia (FH) is the most common inherited cause of hypercholesterolemia and may be one of the most common single-gene disorders in humans. The incidence of the mutation that causes FH is estimated to be as common as 1 in 250 to 1 in 500 individuals in the population. FH is also known as autosomal dominant hypercholesterolemia, type 1, and is caused by loss-of-function mutations in the gene encoding the low-density lipoprotein (LDL) receptor. More than 1600 mutations in the gene have been reported. In the presence of a single mutation (heterozygous FH), there is a decrease of LDL receptors in the liver with a resulting decrease in clearance of LDL from the circulation, and plasma levels of LDL cholesterol typically range from 200–400 mg/dL. In the presence of two mutations (homozygous FH), LDL receptors are markedly reduced or absent. In these patients, the LDL cholesterol levels are markedly elevated to 400 to >1000 mg/dL. Many individuals with homozygous FH present with cutaneous xanthomas in childhood and early cardiovascular disease in late childhood or young adulthood. Although heterozygous patients have hypercholesterolemia from birth, disease recognition is usually not until adulthood when patients are found to have tendon xanthomas or coronary artery disease. In patients with heterozygous disease, there is generally a family history on at least one side of the family. Familial defective apolipoprotein (apo) B-100 has a similar presentation but is less common (1/1000). ApoA-V deficiency presents with xanthomas, but also pancreatitis and hepatosplenomegaly with elevated chylomicrons and VLDL. Familial hepatic lipase deficiency and lipoprotein lipase deficiency are associated with increased chylomicrons, not LDL cholesterol, and present with eruptive xanthomas, hepatosplenomegaly, and pancreatitis. These conditions occur rarely (<1/1,000,000).
++
++
++
All of the following are potential causes of elevated LDL EXCEPT:
++
++
++
++
++
++
++
++
++
++
++
+
++
The answer is B. (Chap. 421) There are many secondary forms of elevated LDL that warrant consideration in a patient found to have abnormal LDL. These include hypothyroidism, nephrotic syndrome, cholestasis, acute intermittent porphyria, anorexia nervosa, hepatoma, and drugs such as thiazides, cyclosporine, and carbamazepine. Cirrhosis is associated with reduced LDL because of inadequate production. Malabsorption, malnutrition, Gaucher disease, chronic infectious disease, hyperthyroidism, and niacin toxicity are all similarly associated with reduced LDL.
++
++
++
Your 60-year-old patient with a monoclonal gammopathy of unclear significance presents for a follow-up visit and to review recent laboratory data. His creatinine is newly elevated to 2.0 mg/dL, potassium is 3.7 mg/dL, calcium is 12.2 mg/dL, LDL is 202 mg/dL, and triglycerides are 209 mg/dL. On further questioning, he reports 3 months of swelling around the eyes and “foamy” urine. On examination, he has anasarca. Concerned for multiple myeloma and nephrotic syndrome, you order a urine protein/creatinine ratio, which returns at 14:1. Which treatment option would be most appropriate to treat his lipid abnormalities?
++
++
++
A. Cholesterol ester transfer protein inhibitor
++
++
++
++
C. 3-Hydroxy-3-methylglutaryl–coenzyme A (HMG-CoA) reductase inhibitors
++
++
++
++
+
++
The answer is C. (Chap. 421) This patient has nephrotic syndrome, which is likely a result of multiple myeloma. The hyperlipidemia of nephrotic syndrome appears to be due to a combination of increased hepatic production and decreased clearance of VLDL, with increased LDL production. It is usually mixed but can manifest as hypercholesterolemia or hypertriglyceridemia. Effective treatment of the underlying renal disease normalizes the lipid profile. Of the choices presented, 3-hydroxy-3-methylglutaryl–coenzyme A (HMG-CoA) reductase inhibitors would be the most effective to reduce this patient’s LDL. Dietary management is an important component of lifestyle modification but seldom results in a >10% fall in LDL. Niacin and fibrates would be indicated if the triglycerides were higher, but the LDL is the more important lipid abnormality to address at this time. Lipid apheresis is reserved for patients who cannot tolerate the lipid-lowering drugs or who have a genetic lipid disorder refractory to medication. Cholesterol ester transfer protein inhibitors have been shown to raise high-density lipoprotein (HDL) levels, and their role in the treatment of lipoproteinemias is still under investigation.
++
++
++
Metabolic syndrome was defined initially as a clinical entity by the World Health Organization in 1998 as a constellation of findings including central obesity, hypertriglyceridemia, low high-density lipoprotein (HDL), hyperglycemia, and hypertension. Which of the following statements regarding the epidemiology of metabolic syndrome is true?
++
++
++
A. After the age of 60, men are more likely to have metabolic syndrome than women.
++
++
B. Among patients with diabetes mellitus, presence of metabolic syndrome confers a higher risk of cardiovascular disease.
++
++
C. BMI is the strongest predictor of insulin resistance and diabetes risk in metabolic syndrome.
++
++
D. The highest recorded prevalence of metabolic syndrome in the United States is among Mexican American women.
++
++
E. The nationality at the lowest risk of metabolic syndrome is the Japanese population.
+
++
The answer is B. (Chap. 422) Metabolic syndrome is a common disorder that features central obesity, hypertriglyceridemia, low levels of HDL cholesterol, hyperglycemia, and hypertension. The prevalence of the disease varies around the world, reflecting the age, ethnicity, and varying diagnostic criteria applied. The highest prevalence of metabolic syndrome worldwide occurs in the Native American populations in the United States, with nearly 60% of women age 45–49 and 45% of men age 45–49 being affected. In the United States, African American men are less commonly affected, whereas Mexican American women are more commonly affected. In France, the disease prevalence is generally the lowest in the world, with <10% of individuals between 30 and 60 years of age affected, although after age 60, the prevalence rises to 17.5%. Risk factors that confer increasing likelihood of developing metabolic syndrome included overweight/obesity, aging, sedentary lifestyle, diabetes mellitus, cardiovascular disease, and lipodystrophy. Central obesity is both a risk factor and a feature central to defining the presence of the disease. Central obesity as measured by waist circumference, not body mass index, is most strongly associated with insulin resistance and risk of diabetes mellitus and cardiovascular disease. The precise waist circumference at which the risk increases may vary between men and women and across different ethnicities. For instance, in Japanese women, the waist circumference that is used for diagnosis of metabolic syndrome is 90 cm compared to 85 cm for men. However, in individuals of Europoid descent, women are diagnosed with metabolic syndrome at a waist circumference ≥80 cm, whereas men are diagnosed at a waist circumference ≥94 cm. Aging is also associated with increased risk of metabolic syndrome. Metabolic syndrome affects about half of the population older than age 50, and after age 60, women are more affected than men. Physical inactivity is a predictor of cardiovascular events and death in individuals with metabolic syndrome. Spending more than 4 hours a day watching television or videos or using a computer confers a twofold greater risk of metabolic syndrome. Insulin resistance is felt to be the pathophysiologic hallmark of the metabolic syndrome, and about 75% of individuals with T2DM or impaired glucose tolerance have metabolic syndrome. When these diseases coexist in an individual, there is a higher prevalence of cardiovascular disease than with T2DM or glucose tolerance alone.
++
++
++
A 47-year-old Chinese man is seen for an annual examination. He generally has no complaints but lives a sedentary lifestyle. He works as an accountant and spends of his work days at a computer screen. He does not maintain a regular exercise routine. He admits his diet is poor. He is divorced and lives alone. He eats out or gets take out about 4 nights a week. On other days, he prefers quick meals that he can heat up in the microwave. His past medical history is significant for hypertension and obesity. He is being treated with hydrochlorothiazide 25 mg daily. He has no allergies. His blood pressure today is 148/92 mmHg. His waist circumference is 93 cm (36.6 in). He is 177.8 cm (70 in) tall and weighs 105 kg (225 lb). His BMI is 32.3 kg/m2. On his annual fasting labs, his total cholesterol is 220 mg/dL, HDL is 28 mg/dL, triglycerides are 178 mg/dL, and LDL is 103 mg/dL. Fasting plasma glucose is 98 mg/dL. Which of the following is true regarding a diagnosis of metabolic syndrome in this patient?
++
++
++
A. He cannot have metabolic syndrome because his fasting plasma glucose level is normal.
++
++
B. He has metabolic syndrome because he meets three out of five diagnostic criteria: high triglyceride levels, low HDL level, and hypertension.
++
++
C. He has metabolic syndrome because he meets four out of five diagnostic criteria: high BMI, high triglyceride levels, low HDL level, and hypertension.
++
++
D. He has metabolic syndrome because he meets four out of five diagnostic criteria: large waist circumference, high triglyceride levels, low HDL level, and hypertension.
++
++
E. Metabolic syndrome cannot be diagnosed on a single evaluation. Repeat testing is indicated in 3–6 months
+
++
The answer is D. (Chap. 422) The most recent criteria for the diagnosis of metabolic syndrome is called the Harmonizing Definition and was published in 2009. This definition brought together multiple international medical societies to create a unifying definition, including the International Diabetes Federation; the National Heart, Lung, and Blood Institute; the American Heart Association; the World Heart Federation; the International Atherosclerosis Society; and the International Association for the Study of Obesity. When compared to prior guidelines, the most important change was to recognize that the waist circumference that confers the risk of metabolic syndrome is different across ethnic groups. The harmonizing definition creates three different waist circumference groupings by gender and ethnic group (Table X-73).
++
++
Furthermore, when compared with the National Cholesterol Education Program and Adult Treatment Panel III 2001 classification, the waist circumference that is considered abnormal is lower by at least 8 cm in both men and women. The remainder of the diagnostic criteria for metabolic syndrome remain the same when compared to prior guidelines:
++
Fasting triglycerides >150 mg/dL or specific medication
HDL cholesterol <40 mg/dL in men or <50 mg/dL in women or specific medication
Blood pressure >130 mmHg systolic or >85 mmHg diastolic or specific medication or previous diagnosis
Fasting plasma glucose >100 mg/dL or drug treatment of elevated glucose level
++
This individual meets diagnostic criteria with elevated waist circumference (>90 cm in Chinese man), elevated triglyceride level, low HDL level, and hypertension.
++
++
++
A 55-year-old man is admitted to the intensive care unit with 1 week of fever and cough. He was well until 1 week before admission, when he noted progressive shortness of breath, cough, and productive sputum. On the day of admission, the patient was noted by his wife to be lethargic. Emergency response medics found the patient unresponsive. He was intubated in the field and brought to the emergency department. His only medications are insulin glargine 20 units daily and insulin aspart with meals. The past medical history is notable for alcohol abuse and diabetes mellitus. His recent alcohol use has been at least 12 beers daily. Upon arrival to the hospital, his temperature is 38.9°C (102°F), blood pressure is 76/40 mmHg, and oxygen saturation is 86% on ventilator setting of assist-control, with a tidal volume of 420 mL, respiratory rate of 22 breaths/min, positive end-expiratory pressure of 5, and FiO2 of 1.0. On examination, the patient is intubated on mechanical ventilation. Jugular venous pressure is normal. There are decreased breath sounds at the right lung base with egophony. Heart sounds are normal. The abdomen is soft. There is no peripheral edema. Chest radiography shows a right lower lobe infiltrate with a moderate pleural effusion. An electrocardiogram is normal. Sputum Gram stain shows gram-positive diplococci. White blood cell count is 23 × 103/μL, with 70% polymorphonuclear cells and 6% bands. Blood urea nitrogen is 80 mg/dL, and creatinine is 3.1 mg/dL. Plasma glucose is 425 mg/dL. He is started on broad-spectrum antibiotics, IV fluids, omeprazole, and an insulin drip. A nasogastric tube is inserted, and tube feedings are started. On hospital day 2, his creatinine has improved to 1.6 mg/dL. However, plasma phosphate is 1.0 mg/dL (0.3 mmol/L), and calcium is 8.8 mg/dL. All of following are causes of hypophosphatemia is this patient EXCEPT:
++
++
++
++
++
++
++
++
++
++
++
+
++
The answer is A. (Chap. 423) Hypophosphatemia results from one of three mechanisms: inadequate intestinal phosphate absorption, excessive renal phosphate excretion, and rapid redistribution of phosphate from the extracellular space into bone or soft tissue. Inadequate intestinal absorption is rare because antacids containing aluminum hydroxide are no longer commonly prescribed. Malnutrition from fasting or starvation may result in depletion of phosphate. This is also commonly seen in alcoholism. In hospitalized patients, redistribution is the main cause. Insulin promotes phosphate entry into cells along with glucose. When nutrition is initiated, refeeding further increases redistribution of phosphate into cells and is more pronounced when IV glucose is used alone. Sepsis may cause destruction of cells and metabolic acidosis, resulting in a net shift of phosphate from the extracellular space into cells. Renal failure is associated with hyperphosphatemia, not hypophosphatemia, and initial prerenal azotemia, such as in this presentation, can obscure underlying phosphate depletion. The approach to treating hypophosphatemia should take into account several factors, including the likelihood (and magnitude) of underlying phosphate depletion, renal function, serum calcium levels, and the concurrent administration of parenteral glucose. In addition, the treating physician should assess the patient for complications of hypophosphatemia, which can include neuromuscular weakness, cardiac dysfunction, hemolysis, and platelet dysfunction. Severe hypophosphatemia generally occurs when the serum concentration falls below 2 mg/dL (<0.75 mmol/L). This becomes particularly dangerous when there is underlying chronic phosphate depletion. However, there is no simple formula to determine the body’s phosphate needs from measurement of the serum phosphate levels because most phosphate is intracellular. It is generally recommended to use oral phosphate repletion when the serum phosphate levels are greater than 1.5–2.5 mg/dL (0.5–0.8 mmol/L). The dose of oral phosphate is 750–2000 mg daily of elemental phosphate given in divided doses. More severe hypophosphatemia, as in the case presented, requires IV repletion. IV phosphate repletion is given as neutral mixtures of sodium and potassium phosphate salts at doses of 0.2–0.8 mmol/kg given over 6 hours. Table X-75 outlines the total dose and recommended infusion rates for a range of phosphate levels. In this patient with a level of 1.0 mg/dL, the recommended infusion rate is 8 mmol/hr over 6 hours for a total dose of 48 mmol. Until the underlying hypophosphatemia is corrected, one should measure phosphate and calcium levels every 6 hours. The infusion should be stopped if the calcium phosphate product rises to higher than 50 to decrease the risk of heterotopic calcification. Alternatively, if hypocalcemia is present coincident with the hypophosphatemia, it is important to correct the calcium prior to administering phosphate.
++
++
++
++
In the patient described, A 55-year-old man is admitted to the intensive care unit with 1 week of fever and cough. He was well until 1 week before admission, when he noted progressive shortness of breath, cough, and productive sputum. On the day of admission, the patient was noted by his wife to be lethargic. Emergency response medics found the patient unresponsive. He was intubated in the field and brought to the emergency department. His only medications are insulin glargine 20 units daily and insulin aspart with meals. The past medical history is notable for alcohol abuse and diabetes mellitus. His recent alcohol use has been at least 12 beers daily. Upon arrival to the hospital, his temperature is 38.9°C (102°F), blood pressure is 76/40 mmHg, and oxygen saturation is 86% on ventilator setting of assist-control, with a tidal volume of 420 mL, respiratory rate of 22 breaths/min, positive end-expiratory pressure of 5, and FiO2 of 1.0. On examination, the patient is intubated on mechanical ventilation. Jugular venous pressure is normal. There are decreased breath sounds at the right lung base with egophony. Heart sounds are normal. The abdomen is soft. There is no peripheral edema. Chest radiography shows a right lower lobe infiltrate with a moderate pleural effusion. An electrocardiogram is normal. Sputum Gram stain shows gram-positive diplococci. White blood cell count is 23 × 103/μL, with 70% polymorphonuclear cells and 6% bands. Blood urea nitrogen is 80 mg/dL, and creatinine is 3.1 mg/dL. Plasma glucose is 425 mg/dL. He is started on broad-spectrum antibiotics, IV fluids, omeprazole, and an insulin drip. A nasogastric tube is inserted, and tube feedings are started. On hospital day 2, his creatinine has improved to 1.6 mg/dL. However, plasma phosphate is 1.0 mg/dL (0.3 mmol/L), and calcium is 8.8 mg/dL, what it the most appropriate approach to correcting the hypophosphatemia?
++
++
++
A. Administer IV calcium gluconate 1 g followed by infusion of IV phosphate at a rate of 8 mmol/hr for 6 hours.
++
++
B. Administer IV phosphate alone at a rate of 2 mmol/hr for 6 hours.
++
++
C. Administer IV phosphate alone at a rate of 8 mmol/hr for 6 hours.
++
++
D. Continued close observation as redistribution of phosphate is expected to normalize levels over the course of the next 24–48 hours.
++
++
E. Initiate oral phosphate replacement at a dose of 1500 mg/d.
+
++
The answer is C. (Chap. 423) Hypophosphatemia results from one of three mechanisms: inadequate intestinal phosphate absorption, excessive renal phosphate excretion, and rapid redistribution of phosphate from the extracellular space into bone or soft tissue. Inadequate intestinal absorption is rare because antacids containing aluminum hydroxide are no longer commonly prescribed. Malnutrition from fasting or starvation may result in depletion of phosphate. This is also commonly seen in alcoholism. In hospitalized patients, redistribution is the main cause. Insulin promotes phosphate entry into cells along with glucose. When nutrition is initiated, refeeding further increases redistribution of phosphate into cells and is more pronounced when IV glucose is used alone. Sepsis may cause destruction of cells and metabolic acidosis, resulting in a net shift of phosphate from the extracellular space into cells. Renal failure is associated with hyperphosphatemia, not hypophosphatemia, and initial prerenal azotemia, such as in this presentation, can obscure underlying phosphate depletion. The approach to treating hypophosphatemia should take into account several factors, including the likelihood (and magnitude) of underlying phosphate depletion, renal function, serum calcium levels, and the concurrent administration of parenteral glucose. In addition, the treating physician should assess the patient for complications of hypophosphatemia, which can include neuromuscular weakness, cardiac dysfunction, hemolysis, and platelet dysfunction. Severe hypophosphatemia generally occurs when the serum concentration falls below 2 mg/dL (<0.75 mmol/L). This becomes particularly dangerous when there is underlying chronic phosphate depletion. However, there is no simple formula to determine the body’s phosphate needs from measurement of the serum phosphate levels because most phosphate is intracellular. It is generally recommended to use oral phosphate repletion when the serum phosphate levels are greater than 1.5–2.5 mg/dL (0.5–0.8 mmol/L). The dose of oral phosphate is 750–2000 mg daily of elemental phosphate given in divided doses. More severe hypophosphatemia, as in the case presented, requires IV repletion. IV phosphate repletion is given as neutral mixtures of sodium and potassium phosphate salts at doses of 0.2–0.8 mmol/kg given over 6 hours. Table X-75 outlines the total dose and recommended infusion rates for a range of phosphate levels. In this patient with a level of 1.0 mg/dL, the recommended infusion rate is 8 mmol/hr over 6 hours for a total dose of 48 mmol. Until the underlying hypophosphatemia is corrected, one should measure phosphate and calcium levels every 6 hours. The infusion should be stopped if the calcium phosphate product rises to higher than 50 to decrease the risk of heterotopic calcification. Alternatively, if hypocalcemia is present coincident with the hypophosphatemia, it is important to correct the calcium prior to administering phosphate.
++
++
++
++
You are caring for a 72-year-old man who has been living in a nursing home for the past 3 years. He has severe chronic obstructive pulmonary disease and requires continuous oxygen at 3 L/min. He also previously had a stroke, which has left him with a right hemiparesis. His current medications include aspirin, losartan, hydrochlorothiazide, fluticasone/salmeterol, tiotropium, and albuterol. His BMI is 18.5 kg/m2. You are concerned that he may have vitamin D deficiency. Which of the following is the best test to determine if vitamin D deficiency is present?
++
++
++
A. 1,25-Hydroxy-vitamin D
++
++
++
++
++
++
++
++
E. Serum total and ionized calcium levels
+
++
The answer is B. (Chap. 423) Vitamin D deficiency is highly prevalent in the United States and is most common in older individuals who are hospitalized or institutionalized. Vitamin D deficiency can occur as a result of inadequate dietary intake, decreased production in the skin, decreased intestinal absorption, accelerated losses, or impaired vitamin D activation in the liver or kidney. Clinically, vitamin D deficiency in older individuals is most often silent. Often practitioners fail to consider vitamin D deficiency until a patient has been diagnosed with osteoporosis or suffered a fracture. However, some individuals can experience diffuse muscle and bone pain. When assessing vitamin D levels, the appropriate test is 25-hydroxyvitamin D [25(OH)D] levels. The Institute of Medicine has defined vitamin D sufficiency as a level of 25(OH)D >50 nmol/L (>20 ng/L). However, in the elderly and in some disease states, higher levels may be required to maximize intestinal calcium absorption. Levels less than 37 nmol/L (15 ng/mL) are associated with a rise in parathyroid hormone levels and a fall in bone density. Vitamin D deficiency may also lead to decreased intestinal absorption of calcium with resultant hypocalcemia and secondary hyperparathyroidism. As a result, there is higher bone turnover, which can be associated with an increase in alkaline phosphatase levels. In addition, elevated parathyroid hormone (PTH) stimulates renal conversion of 25(OH)D to 1,25-hydroxyvitamin D [1,25(OH)D], the activated form of vitamin D. Thus, even in the face of severe vitamin D deficiency, the activated 1,25(OH)D levels may be normal and do not accurately reflect vitamin D stores. Thus, 1,25(OH)D should not be used to make a diagnosis of vitamin D deficiency. Although vitamin D deficiency may be associated with abnormalities in PTH, alkaline phosphatase, and calcium levels, these biochemical abnormalities are seen in many other diseases and are neither sensitive nor specific for the diagnosis of vitamin D deficiency.
++
++
++
A 72-year-old woman was hospitalized with a right hip fracture. After initial surgical repair, she is transferred to rehabilitation for further care. While there, she has a 25-hydroxyvitamin D level checked, and it returns at 18.3 ng/L. What do you recommend for treatment in this patient?
++
++
++
A. Vitamin D3 800 units daily
++
++
B. Vitamin D3 800 units daily plus calcium carbonate 1500 mg daily
++
++
C. Vitamin D3 2000 units daily
++
++
D. Vitamin D3 2000 units daily plus calcium carbonate 1500 mg daily
++
++
E. Vitamin D3 50,000 units weekly for 4 weeks, then 800 units weekly, plus calcium 1500 mg daily
+
++
The answer is D. (Chap. 423) Vitamin D deficiency is common in all areas of the United States and has resulted from decreased solar exposure with deficient production of vitamin D in the skin, lack of dietary intake, accelerated losses of vitamin D, impaired vitamin D activation, or resistance to the biologic effects of 1,25(OH)2D, the activated form of vitamin D. Vitamin D stores are best assessed by measuring 25(OH)D. Levels less than 20 ng/L (<50 nmol/L) should be repleted. The recommended daily intake in the absence of vitamin D deficiency is 800 IU of vitamin D, typically administered as vitamin D3 or cholecalciferol, daily. However, higher doses of vitamin D are required when vitamin D deficiency is present to return vitamin D levels to normal. In most individuals, supplementation with vitamin D3 at 2000 IU daily along with calcium supplementation would be recommended. In severe cases of vitamin D deficiency, high-dose repletion may be required. This is given as ergocalciferol (vitamin D) 50,000 IU weekly for 3 to 12 weeks before decreasing to the maintenance daily dose of cholecalciferol 800 IU daily.
++
++
++
A 60-year-old woman is referred to your office for evaluation of hypercalcemia. A serum calcium level of 12.9 mg/dL was found incidentally on a chemistry panel that was drawn during a hospitalization for cholecystectomy. Despite fluid administration in the hospital, her serum calcium at discharge was 11.8 mg/dL. The patient is asymptomatic, and her parathyroid hormone level is 95 ng/L (reference value 10–65 ng/L). She is otherwise in good health and has had her recommended age-appropriate cancer screening. She denies constipation or bone pain and is now 8 weeks out from her surgical procedure. Today, her serum calcium level is 12.6 mg/dL, and phosphate is 2.3 mg/dL. Her hematocrit and all other chemistries, including creatinine, were normal. Which of the following would be an indication for surgery in this patient to definitively treat her underlying diagnosis?
++
++
++
++
++
B. Elevated 24-hour urine calcium
++
++
++
++
D. Osteopenia on bone density testing
++
++
E. Serum calcium >1 mg/dL above normal
+
++
The answer is E. (Chap. 424) Primary hyperparathyroidism is the most common cause of hypercalcemia and is the most likely cause in an adult who is asymptomatic. Primary hyperparathyroidism results from autonomous secretion of PTH that is no longer regulated by serum calcium levels, usually related to development of parathyroid adenomas. Most patients are asymptomatic or have minimal symptoms at the time of diagnosis. When present, symptoms include recurrent nephrolithiasis, peptic ulcers, dehydration, constipation, and altered mental status. Distinctive bone manifestations include osteitis fibrosa cystica, which histologically results from an increase in the giant multinucleated osteoclasts in scalloped areas on the surface of the bone and a replacement of the normal cellular and marrow elements by fibrous tissue. On x-ray, this will appear as resorption of the phalangeal tufts and replacement of the usually sharp cortical outline of the bone in the digits by an irregular outline. Historically, this finding was present on presentation in 10%–25% of cases but is rare today due to earlier diagnosis of disease. Laboratory studies show elevated serum calcium with decreased serum phosphate. Diagnosis can be confirmed with measurement of PTH levels. The optimal management of asymptomatic primary hyperparathyroidism has been debated because surgical removal of autonomous adenomas is generally curative. However, it is unclear whether all patients need to be treated surgically. The most recent recommendations suggest that the more aggressive surgical approach be considered in most patients due to concerns of subtle neuropsychiatric symptoms, long-term skeletal effects, and potential for cardiovascular deterioration. The current guidelines recommend surgery for individuals less than 50 years in age or with creatinine clearance <60 mL/min, osteoporosis on bone density scanning, or serum calcium >1 mg/mL above normal. There is no indication for surgery based on 24-hour urine calcium levels or presence of nephrolithiasis. Likewise, presence of cardiovascular disease is not in the guidelines for recommendation of surgical intervention.
++
++
++
A 42-year-old man presents to the emergency department with acute-onset right-sided flank pain. He describes the pain as 10 out of 10 in severity and radiating to the groin. He has had one episode of hematuria. A noncontrast CT scan confirms the presence of a right-sided renal stone that is currently located in the distal ureter. He has a past medical history of pulmonary sarcoidosis that is not currently treated. This was diagnosed by bronchoscopic biopsy showing noncaseating granulomas. His chest radiograph shows bilateral hilar adenopathy. His serum calcium level is 12.6 mg/dL. What is the mechanism of hypercalcemia in this patient?
++
++
++
A. Increased activation of 25-hydroxyvitamin D to 1,25-hydroxyvitamin D by macrophages within granulomas
++
++
B. Increased activation of 25-hydroxyvitamin D to 1,25-hydroxyvitamin D by the kidney
++
++
C. Increased activation of vitamin D to 25-hydroxyvitamin D by macrophages within granulomas
++
++
D. Missed diagnosis of lymphoma with subsequent bone marrow invasion and resorption of bone through local destruction
++
++
E. Production of parathyroid hormone–related peptide by macrophages within granulomas
+
++
The answer is A. (Chap. 424) Granulomatous disorders including sarcoidosis, tuberculosis, and fungal infections can be associated with hypercalcemia caused by increased conversion of 25(OH)D to 1,25(OH)D by macrophages within the granulomas. This process bypasses the normal feedback mechanisms, and elevated levels of both 25(OH)D and 1,25(OH)D can be seen. This does not normally occur because 1,25(OH)D levels are normally tightly controlled through feedback mechanisms on renal 1-α-hydroxylase, the primary producer of activated vitamin D in normal circumstances. In addition, the normal feedback provided by PTH concentrations is also bypassed and the PTH level may be low.
++
++
++
A 52-year-old man has end-stage kidney disease from longstanding hypertension and diabetes mellitus. He has been managed with hemodialysis for the past 8 years. Throughout this time, he has been poorly compliant with his medications and hemodialysis schedule, frequently missing one session weekly. He is now complaining of bone pain and dyspnea. His oxygen saturation is noted to be 92% on room air, and his chest radiograph shows hazy bilateral infiltrates. Chest CT shows ground-glass infiltrates bilaterally. His laboratory data include a calcium of 12.3 mg/dL, phosphate of 8.1 mg/dL, and parathyroid hormone of 110 pg/mL. Which of the following would be the best approach to the treatment of the patient’s current clinical condition?
++
++
++
A. Calcitriol 0.5 μg IV with hemodialysis with sevelamer three times daily
++
++
B. Calcitriol 0.5 μg orally daily with sevelamer 1600 mg three times daily
++
++
C. More aggressive hemodialysis to achieve optimal fluid and electrolyte balance
++
++
++
++
E. Sevelamer 1600 mg three times daily
+
++
The answer is D. (Chap. 424) This patient demonstrates evidence of tertiary hyperparathyroidism, with inappropriate elevations in PTH despite increases in calcium and phosphate. In addition, the patient is demonstrating clinical evidence of disease including bony pain and ectopic calcification. Tertiary hyperparathyroidism most commonly develops in individuals with longstanding renal failure who have been nonadherent to therapy. In this patient scenario, the hypoxemia and ground-glass infiltrates on chest CT represent ectopic calcification of the lungs. This can be difficult to identify with typical imaging, and a technetium-99 bone scan will show increased uptake in the lungs. Treatment of tertiary hyperparathyroidism with severe clinical manifestations requires parathyroidectomy. Pathologically, these individuals demonstrate the emergence of monoclonal growth in one or more previously hyperplastic parathyroid glands with subsequent autonomous parathyroid function.
++
++
++
A 54-year-old woman undergoes total thyroidectomy for follicular carcinoma of the thyroid. About 6 hours after surgery, the patient complains of tingling around her mouth. She subsequently develops a pins-and-needles sensation in the fingers and toes. The nurse calls the physician to the bedside to evaluate the patient after she has severe hand cramps when her blood pressure is taken. Upon evaluation, the patient is still complaining of intermittent cramping of her hands. Since surgery, she has received morphine sulfate for pain and metoclopramide for nausea. She has had no change in her vital signs and is afebrile. Tapping on the inferior portion of the zygomatic arch 2 cm anterior to the ear produces twitching at the corner of the mouth. An electrocardiogram shows a QT interval of 575 msec. What is the next step in evaluation and treatment of this patient?
++
++
++
A. Administration of benztropine
++
++
B. Administration of calcium gluconate
++
++
C. Administration of magnesium sulphate
++
++
D. Measurement of calcium, magnesium, phosphate, and potassium levels
++
++
E. Measurement of forced vital capacity
+
++
The answer is B. (Chap. 424) Hypocalcemia can be a life-threatening consequence of thyroidectomy if the parathyroid glands are inadvertently removed during the surgery, because the four parathyroid glands are located immediately posterior to the thyroid gland. This currently occurs infrequently because the parathyroid glands are better able to be identified both before and during surgery. However, hypoparathyroidism may occur even if the parathyroid glands are not removed by thyroidectomy due to devascularization or trauma to the parathyroid glands. Hypocalcemia following removal of the parathyroid glands may begin any time during the first 24–72 hours, and monitoring of serial calcium levels is recommended for the first 72 hours. The earliest symptoms of hypocalcemia are typically circumoral paresthesias and paresthesias with a “pins-and-needles” sensation in the fingers and toes. The development of carpal spasms upon inflation of the blood pressure cuff is a classic sign of hypocalcemia and is known as Trousseau sign. Chvostek sign is the other classic sign of hypocalcemia and is elicited by tapping the facial nerve in the preauricular area causing spasm of the facial muscles. A prolongation of the QT interval on the electrocardiogram suggests life-threatening hypocalcemia that may progress to fatal arrhythmia, and treatment should not be delayed while waiting for serum testing to occur in a patient with a known cause of hypocalcemia. Immediate treatment with IV calcium should be initiated. Maintenance therapy with calcitriol and vitamin D is necessary for ongoing treatment of acquired hypoparathyroidism. Alternatively, surgeons may implant parathyroid tissue into the soft tissue of the forearm, if it is thought that the parathyroid glands will be removed. Hypomagnesemia can cause hypocalcemia by suppressing PTH release despite the presence of hypocalcemia. However, in this patient, hypomagnesemia is not suspected after thyroidectomy, and magnesium administration is not indicated. Benztropine is a centrally acting anticholinergic medication that is used in the treatment of dystonic reactions that can occur after taking centrally acting antiemetic medications with dopaminergic activity, such as metoclopramide or Compazine. Dystonic reactions involve focal spasms of the face, neck, and extremities. Although this patient has taken medications that can cause a dystonic reaction, the spasms that she is experiencing are more consistent with tetanic contractions of hypocalcemia than dystonic reaction. Finally, measurement of forced vital capacity is most commonly used as a measurement of disease severity in myasthenia gravis or Guillain-Barré syndrome. Muscle weakness is a typical presenting feature, but not paresthesias.
++
++
++
A 68-year-old woman with stage IIIB squamous cell carcinoma of the lung is admitted to the hospital because of altered mental status and dehydration. Upon admission, she is found to have a calcium level of 19.6 mg/dL and phosphate level of 1.8 mg/dL. Concomitant measurement of parathyroid hormone was 0.1 pg/mL (normal 10–65 pg/mL), and a screen for parathyroid hormone–related peptide was positive. Over the first 24 hours, the patient receives 4 L of normal saline with furosemide diuresis. The next morning, the patient’s calcium is 17.6 mg/dL and phosphate is 2.2 mg/dL. She continues to have delirium. What is the best approach for ongoing treatment of this patient’s hypercalcemia?
++
++
++
A. Continue therapy with large-volume fluid administration and forced diuresis with furosemide
++
++
B. Continue therapy with large-volume fluid administration, but stop furosemide and treat with hydrochlorothiazide
++
++
C. Initiate therapy with calcitonin alone
++
++
D. Initiate therapy with pamidronate alone
++
++
E. Initiate therapy with calcitonin and pamidronate
+
++
The answer is E. (Chap. 424) Malignancy can cause hypercalcemia by several different mechanisms, including metastasis to bone, cytokine stimulation of bone turnover, and production of a protein structurally similar to PTH by the tumor. This protein is called PTH-related peptide (PTHrp) and acts at the same receptors as PTH. Squamous cell carcinoma of the lung is the most common tumor associated with the production of PTHrp. Serum calcium levels can become quite high in malignancy because of unregulated production of PTHrp that is outside of the negative feedback control that normally results in the setting of hypercalcemia. PTH hormone levels should be quite low or undetectable in this setting. When hypercalcemia is severe (>15 mg/dL), symptoms frequently include dehydration and altered mental status. The electrocardiogram may show a shortened QTc interval. Initial therapy includes large-volume fluid administration to reverse the dehydration that results from hypercalciuria. In addition, furosemide is also added to promote further calciuria. If the calcium remains elevated, as in this patient, additional measures should be undertaken to decrease the serum calcium. Calcitonin has a rapid onset of action, with a decrease in serum calcium seen within hours. However, tachyphylaxis develops, and the duration of benefit is limited. Pamidronate is a bisphosphonate that is useful for the hypercalcemia of malignancy. It decreases serum calcium by preventing bone resorption and release of calcium from the bone. After IV administration, the onset of action of pamidronate is 1–2 days, with a duration of action of several weeks. Thus, in this patient with ongoing severe symptomatic hypercalcemia, addition of both calcitonin and pamidronate is the best treatment. The patient should continue to receive IV fluids and furosemide. The addition of a thiazide diuretic is contraindicated because thiazides cause increased calcium resorption in the kidney and would worsen hypercalcemia.
++
++
++
Which of the following statements regarding the epidemiology of osteoporosis and bone fractures is correct?
++
++
++
A. For every 5-year age increase after age 70, the incidence of hip fractures increases by 25%.
++
++
B. Fractures of the distal radius increase in frequency before age 50 and plateau by age 60, with only a modest age-related increase.
++
++
C. Most women meet the diagnostic criteria for osteoporosis between the ages of 60 and 70.
++
++
D. The risk of hip fracture is equal when white women are compared to black women.
++
++
E. Women outnumber men with osteoporosis at a ratio of about 10 to 1.
+
++
The answer is B. (Chap. 425) Osteoporosis refers to a chronic condition characterized by decreased bone strength and frequently manifests as vertebral and hip fractures. In the United States, about 8 million women have osteoporosis compared to about 2 million men, for a ratio in women to men of 4 to 1. An additional 48 million individuals are estimated to have osteopenia. The risk of osteoporosis increases with advancing age and rapidly worsens following menopause in women. Most women meet the diagnostic criteria for osteoporosis between the ages of 70 and 80. White women have an increased risk for osteoporosis when compared to African American women. The epidemiology for bone fractures follows the epidemiology for osteoporosis. Fractures of the distal radius (Colles fracture) increase up to age 50 and plateau by age 60, and there is only a modest increase in risk thereafter. This is contrasted with the risk of hip fractures. Incidence rates for hip fractures double every 5 years after the age of 70. This change in fracture pattern is not entirely due to osteoporosis but is also related to the fact that fewer falls in the elderly occur onto an outstretched arm and are more likely to occur directly on the hip. Black women experience hip fractures at approximately half the rate as white women. The mortality rate in the year following a hip fracture is 5%–20%. Vertebral fractures are also common manifestations of osteoporosis. Although most are found incidentally on chest radiograph, severe cases can lead to height loss, pulmonary restriction, and respiratory morbidity.
++
++
++
A 50-year-old woman presents to your office to inquire about her risk of fracture related to osteoporosis. She has a positive family history of osteoporosis in her mother, but her mother never experienced any hip or vertebral fractures. The patient herself has also not experienced any fractures. She is white and has a 20-pack-year history of tobacco use, quitting 10 years prior. At the age of 37, she had a total hysterectomy with bilateral salpingo-oophorectomy for endometriosis. She is lactose intolerant and does not consume dairy products. She currently takes calcium carbonate 500 mg daily. Her weight is 115 lb, and her height is 66 in (BMI 18.6 kg/m2). All of the following are risk factors for an osteoporotic fracture in this woman EXCEPT:
++
++
++
++
++
++
++
C. History of cigarette smoking
++
++
++
++
+
++
The answer is C. (Chap. 425) There are multiple risks for osteoporotic bone fractures that can be either modifiable or nonmodifiable. These are outlined in Table X-84. Nonmodifiable risk factors include a previous history of fracture as an adult, female sex, white race, dementia, advanced age, and history of fracture (but not osteoporosis) in a first-degree relative. Risk factors that are potentially modifiable include low calcium intake, alcoholism, impaired eyesight, recurrent falls, inadequate physical activity, poor health, and estrogen deficiency, including menopause prior to age 45 or prolonged premenstrual amenorrhea. Excessive thinness and low body weight are also risk factors for osteoporosis, although the osteoporosis guidelines do not clearly delineate what is considered excessive thinness. Current cigarette smoking is a risk factor for osteoporosis-related fracture, whereas a prior history of cigarette use is not.
++
++
++
++
A 54-year-old woman is referred to the endocrinology clinic for evaluation of osteoporosis after a recent examination for back pain revealed a compression fracture of the T4 vertebral body. She is perimenopausal with irregular menstrual periods and frequent hot flashes. She does not smoke. She otherwise is well and healthy. Her weight is 70 kg, and height is 168 cm. She has lost 5 cm from her maximum height. A bone mineral density scan shows a T-score of –3.5 standard deviation (SD) and a Z-score of –2.5 SD. All of the following tests are indicated for the evaluation of osteoporosis in this patient EXCEPT:
++
++
++
++
++
++
++
++
++
++
++
E. Vitamin D levels (25-hydroxyvitamin D)
+
++
The answer is B. (Chap. 425) Osteoporosis is a common disease affecting 8 million women and 2 million men in the United States. It is most common in postmenopausal women, but the incidence is also increasing in men. Estrogen loss probably causes bone loss by activation of bone remodeling sites and exaggeration of the imbalance between bone formation and resorption. Osteoporosis is diagnosed by bone mineral density scan. Dual-energy x-ray absorptiometry (DXA) is the most accurate test for measuring bone mineral density. Clinical determinations of bone density are most commonly measured at the lumbar spine and hip. In the DXA technique, two x-ray energies are used to measure the area of the mineralized tissues, and results are compared to gender- and race-matched normative values. The T-score compares an individual’s results to a young population, whereas the Z-score compares the individual’s results to an age-matched population. Osteoporosis is diagnosed when the T-score is –2.5 standard deviations (SD) in the lumbar spine, femoral neck, or total hip. An evaluation for secondary causes of osteoporosis should be considered in individuals presenting with osteoporotic fractures at a young age and those who have very low Z-scores. Initial evaluation should include serum and 24-hour urine calcium levels, renal function panel, hepatic function panel, serum phosphorous level, and vitamin D levels. Other endocrine abnormalities including hyperthyroidism and hyperparathyroidism should be evaluated, and urinary cortisol levels should be checked if there is a clinical suspicion for Cushing syndrome. FSH and LH levels would be elevated but are not useful in this individual because she presents with a known perimenopausal state.
++
++
++
A 45-year-old white woman seeks advice from her primary care physician regarding her risk for osteoporosis and the need for bone density screening. She is a lifelong nonsmoker and drinks alcohol only socially. She has a history of moderate-persistent asthma since adolescence. She is currently on fluticasone, 44 mg/puff twice daily, with good control currently. She last required oral prednisone therapy about 6 months ago when she had influenza that was complicated by an asthma flare. She took prednisone for a total of 14 days. She has had three pregnancies and two live births at ages 39 and 41. She currently has irregular periods occurring approximately every 42 days. Her FSH level is 25 mIU/L and 17β-estradiol level is 115 pg/mL on day 12 of her menstrual cycle. Her mother and maternal aunt both have been diagnosed with osteoporosis. Her mother also has rheumatoid arthritis and requires prednisone therapy, 5 mg daily. Her mother developed a compression fracture of the lumbar spine at age 68. On physical examination, the patient appears well and healthy. Her height is 168 cm. Her weight is 66.4 kg. The chest, cardiac, abdominal, muscular, and neurologic examinations are normal. What do you tell the patient about the need for bone density screening?
++
++
++
A. Because she is currently perimenopausal, she should have a bone density screen every other year until she completes menopause and then have bone densitometry measured yearly thereafter.
++
++
B. Because of her family history, she should initiate bone density screening yearly beginning now.
++
++
C. Bone densitometry screening is not recommended until after completion of menopause.
++
++
D. Delayed childbearing until the fourth and fifth decade decreases her risk of developing osteoporosis, so bone densitometry is not recommended.
++
++
E. Her use of low-dose inhaled glucocorticoids increases her risk of osteoporosis threefold, and she should undergo yearly bone density screening.
+
++
The answer is C. (Chap. 425) Determination of when to initiate screening for osteoporosis with bone densitometry testing can be complicated by multiple factors. In general, most women do not require screening for osteoporosis until after completion of menopause unless there have been unexplained fractures or other risk factors that would suggest osteoporosis. There is no benefit to initiating screening for osteoporosis in the perimenopausal period. Indeed, most expert recommendations do not recommend routine screening for osteoporosis until age 65 or older unless risk factors are present. Risk factors for osteoporosis include advanced age, current cigarette smoking, low body weight (<57.7 kg), family history of hip fracture, and long-term glucocorticoid use. Inhaled glucocorticoids may cause increased loss of bone density, but because this patient is on a low dose of inhaled fluticasone and is not estrogen deficient, bone mineral densitometry cannot be recommended at this time. The risk of osteoporosis related to inhaled glucocorticoids is not well defined, but most studies suggest that the risk is relatively low. Delaying childbearing until the fourth and fifth decades does increase the risk of osteoporosis but does not cause early onset of osteoporosis prior to completion of menopause. The patient’s family history of osteoporosis likewise is not an indication for early screening for osteoporosis.
++
++
++
Match the following medications used for osteoporosis to the mechanism of action:
++
++
++
++
A. Recombinant parathyroid hormone (1-34hPTH) with direct stimulation of osteoblast activity
++
++
B. Polypeptide hormone that suppresses osteoclast activity through a specific receptor for the hormone
++
++
C. Bisphosphonate drug given on an annual basis that impairs osteoclast function and reduces osteoclast number
++
++
D. Selective estrogen receptor modulator
++
++
E. Human monoclonal antibody to RANKL, a protein necessary for osteoclast maturation
+
++
The answer is B. (Chap. 425) In the past 20 years, multiple pharmacologic options have become available for the treatment of osteoporosis. Prior to the 1990s, estrogen, either alone or in combination with a progestin, was the primary treatment for osteoporosis. Since that time, many new agents have been introduced, although estrogen is effective at preventing bone loss and reducing bone turnover and yields small increases in bone mass of the spine, hip, and total body. The selective estrogen receptor modulator (SERM) raloxifene binds to the estrogen receptor and is approved for the prevention and treatment of osteoporosis as well as the prevention of breast cancer. Tamoxifen is another well-known SERM, but it is only approved for the treatment and prevention of breast cancer. Both drugs have a favorable effect on bone turnover and bone mass. Raloxifene has been demonstrated in clinical trials to reduce the occurrence of vertebral fracture by 30%–50%, although the effect on nonvertebral fracture is not known. Bisphosphonates are the most widely used category of medications for the prevention and treatment of osteoporosis. Alendronate, risedronate, ibandronate, and zoledronic acid are approved medications in this class. Bisphosphonates act to impair osteoclast function and reduce osteoclast number by inducing apoptosis. Zoledronic acid is retained in the bone for a very long time and is dosed intravenously only once yearly. Exogenous administration of calcitonin, a polypeptide hormone produced by the thyroid gland, is sometimes administered as a nasal spray in the treatment of osteoporosis. It acts to suppress osteoclast activity by direct action on the osteoclast calcitonin receptor. In clinical studies, the effect on bone mass and vertebral fracture risk was small, and there was no effect on nonvertebral fractures. Denosumab is a fully human monoclonal antibody to RANKL, the final common effector of osteoclast formation, activity, and survival. When denosumab binds to RANKL, osteoclast maturation is significantly impaired. It is administered by subcutaneous injection twice yearly and has been demonstrated to decrease fracture risk in the spine, hip, and forearm over a 3-year period by 20%–70%. Teriparatide is a recombinant PTH (1-34hPTH) that is approved for the treatment of osteoporosis. It is administered by daily subcutaneous injection and has been shown to decrease both vertebral and nonvertebral fracture risk. Because teriparatide is an analogue of PTH, the drug acts like PTH with direct actions on osteoblast to stimulate new bone formation, which is unique among the treatments for osteoporosis.
++
++
++
Match the following medications used for osteoporosis to the mechanism of action:
++
++
++
++
A. Recombinant parathyroid hormone (1-34hPTH) with direct stimulation of osteoblast activity
++
++
B. Polypeptide hormone that suppresses osteoclast activity through a specific receptor for the hormone
++
++
C. Bisphosphonate drug given on an annual basis that impairs osteoclast function and reduces osteoclast number
++
++
D. Selective estrogen receptor modulator
++
++
E. Human monoclonal antibody to RANKL, a protein necessary for osteoclast maturation
+
++
The answer is E. (Chap. 425) In the past 20 years, multiple pharmacologic options have become available for the treatment of osteoporosis. Prior to the 1990s, estrogen, either alone or in combination with a progestin, was the primary treatment for osteoporosis. Since that time, many new agents have been introduced, although estrogen is effective at preventing bone loss and reducing bone turnover and yields small increases in bone mass of the spine, hip, and total body. The selective estrogen receptor modulator (SERM) raloxifene binds to the estrogen receptor and is approved for the prevention and treatment of osteoporosis as well as the prevention of breast cancer. Tamoxifen is another well-known SERM, but it is only approved for the treatment and prevention of breast cancer. Both drugs have a favorable effect on bone turnover and bone mass. Raloxifene has been demonstrated in clinical trials to reduce the occurrence of vertebral fracture by 30%–50%, although the effect on nonvertebral fracture is not known. Bisphosphonates are the most widely used category of medications for the prevention and treatment of osteoporosis. Alendronate, risedronate, ibandronate, and zoledronic acid are approved medications in this class. Bisphosphonates act to impair osteoclast function and reduce osteoclast number by inducing apoptosis. Zoledronic acid is retained in the bone for a very long time and is dosed intravenously only once yearly. Exogenous administration of calcitonin, a polypeptide hormone produced by the thyroid gland, is sometimes administered as a nasal spray in the treatment of osteoporosis. It acts to suppress osteoclast activity by direct action on the osteoclast calcitonin receptor. In clinical studies, the effect on bone mass and vertebral fracture risk was small, and there was no effect on nonvertebral fractures. Denosumab is a fully human monoclonal antibody to RANKL, the final common effector of osteoclast formation, activity, and survival. When denosumab binds to RANKL, osteoclast maturation is significantly impaired. It is administered by subcutaneous injection twice yearly and has been demonstrated to decrease fracture risk in the spine, hip, and forearm over a 3-year period by 20%–70%. Teriparatide is a recombinant PTH (1-34hPTH) that is approved for the treatment of osteoporosis. It is administered by daily subcutaneous injection and has been shown to decrease both vertebral and nonvertebral fracture risk. Because teriparatide is an analogue of PTH, the drug acts like PTH with direct actions on osteoblast to stimulate new bone formation, which is unique among the treatments for osteoporosis.
++
++
++
Match the following medications used for osteoporosis to the mechanism of action:
++
++
++
++
A. Recombinant parathyroid hormone (1-34hPTH) with direct stimulation of osteoblast activity
++
++
B. Polypeptide hormone that suppresses osteoclast activity through a specific receptor for the hormone
++
++
C. Bisphosphonate drug given on an annual basis that impairs osteoclast function and reduces osteoclast number
++
++
D. Selective estrogen receptor modulator
++
++
E. Human monoclonal antibody to RANKL, a protein necessary for osteoclast maturation
+
++
The answer is D. (Chap. 425) In the past 20 years, multiple pharmacologic options have become available for the treatment of osteoporosis. Prior to the 1990s, estrogen, either alone or in combination with a progestin, was the primary treatment for osteoporosis. Since that time, many new agents have been introduced, although estrogen is effective at preventing bone loss and reducing bone turnover and yields small increases in bone mass of the spine, hip, and total body. The selective estrogen receptor modulator (SERM) raloxifene binds to the estrogen receptor and is approved for the prevention and treatment of osteoporosis as well as the prevention of breast cancer. Tamoxifen is another well-known SERM, but it is only approved for the treatment and prevention of breast cancer. Both drugs have a favorable effect on bone turnover and bone mass. Raloxifene has been demonstrated in clinical trials to reduce the occurrence of vertebral fracture by 30%–50%, although the effect on nonvertebral fracture is not known. Bisphosphonates are the most widely used category of medications for the prevention and treatment of osteoporosis. Alendronate, risedronate, ibandronate, and zoledronic acid are approved medications in this class. Bisphosphonates act to impair osteoclast function and reduce osteoclast number by inducing apoptosis. Zoledronic acid is retained in the bone for a very long time and is dosed intravenously only once yearly. Exogenous administration of calcitonin, a polypeptide hormone produced by the thyroid gland, is sometimes administered as a nasal spray in the treatment of osteoporosis. It acts to suppress osteoclast activity by direct action on the osteoclast calcitonin receptor. In clinical studies, the effect on bone mass and vertebral fracture risk was small, and there was no effect on nonvertebral fractures. Denosumab is a fully human monoclonal antibody to RANKL, the final common effector of osteoclast formation, activity, and survival. When denosumab binds to RANKL, osteoclast maturation is significantly impaired. It is administered by subcutaneous injection twice yearly and has been demonstrated to decrease fracture risk in the spine, hip, and forearm over a 3-year period by 20%–70%. Teriparatide is a recombinant PTH (1-34hPTH) that is approved for the treatment of osteoporosis. It is administered by daily subcutaneous injection and has been shown to decrease both vertebral and nonvertebral fracture risk. Because teriparatide is an analogue of PTH, the drug acts like PTH with direct actions on osteoblast to stimulate new bone formation, which is unique among the treatments for osteoporosis.
++
++
++
Match the following medications used for osteoporosis to the mechanism of action:
++
++
++
++
A. Recombinant parathyroid hormone (1-34hPTH) with direct stimulation of osteoblast activity
++
++
B. Polypeptide hormone that suppresses osteoclast activity through a specific receptor for the hormone
++
++
C. Bisphosphonate drug given on an annual basis that impairs osteoclast function and reduces osteoclast number
++
++
D. Selective estrogen receptor modulator
++
++
E. Human monoclonal antibody to RANKL, a protein necessary for osteoclast maturation
+
++
The answer is A. (Chap. 425) In the past 20 years, multiple pharmacologic options have become available for the treatment of osteoporosis. Prior to the 1990s, estrogen, either alone or in combination with a progestin, was the primary treatment for osteoporosis. Since that time, many new agents have been introduced, although estrogen is effective at preventing bone loss and reducing bone turnover and yields small increases in bone mass of the spine, hip, and total body. The selective estrogen receptor modulator (SERM) raloxifene binds to the estrogen receptor and is approved for the prevention and treatment of osteoporosis as well as the prevention of breast cancer. Tamoxifen is another well-known SERM, but it is only approved for the treatment and prevention of breast cancer. Both drugs have a favorable effect on bone turnover and bone mass. Raloxifene has been demonstrated in clinical trials to reduce the occurrence of vertebral fracture by 30%–50%, although the effect on nonvertebral fracture is not known. Bisphosphonates are the most widely used category of medications for the prevention and treatment of osteoporosis. Alendronate, risedronate, ibandronate, and zoledronic acid are approved medications in this class. Bisphosphonates act to impair osteoclast function and reduce osteoclast number by inducing apoptosis. Zoledronic acid is retained in the bone for a very long time and is dosed intravenously only once yearly. Exogenous administration of calcitonin, a polypeptide hormone produced by the thyroid gland, is sometimes administered as a nasal spray in the treatment of osteoporosis. It acts to suppress osteoclast activity by direct action on the osteoclast calcitonin receptor. In clinical studies, the effect on bone mass and vertebral fracture risk was small, and there was no effect on nonvertebral fractures. Denosumab is a fully human monoclonal antibody to RANKL, the final common effector of osteoclast formation, activity, and survival. When denosumab binds to RANKL, osteoclast maturation is significantly impaired. It is administered by subcutaneous injection twice yearly and has been demonstrated to decrease fracture risk in the spine, hip, and forearm over a 3-year period by 20%–70%. Teriparatide is a recombinant PTH (1-34hPTH) that is approved for the treatment of osteoporosis. It is administered by daily subcutaneous injection and has been shown to decrease both vertebral and nonvertebral fracture risk. Because teriparatide is an analogue of PTH, the drug acts like PTH with direct actions on osteoblast to stimulate new bone formation, which is unique among the treatments for osteoporosis.
++
++
++
Match the following medications used for osteoporosis to the mechanism of action:
++
++
++
++
A. Recombinant parathyroid hormone (1-34hPTH) with direct stimulation of osteoblast activity
++
++
B. Polypeptide hormone that suppresses osteoclast activity through a specific receptor for the hormone
++
++
C. Bisphosphonate drug given on an annual basis that impairs osteoclast function and reduces osteoclast number
++
++
D. Selective estrogen receptor modulator
++
++
E. Human monoclonal antibody to RANKL, a protein necessary for osteoclast maturation
+
++
The answer is C. (Chap. 425) In the past 20 years, multiple pharmacologic options have become available for the treatment of osteoporosis. Prior to the 1990s, estrogen, either alone or in combination with a progestin, was the primary treatment for osteoporosis. Since that time, many new agents have been introduced, although estrogen is effective at preventing bone loss and reducing bone turnover and yields small increases in bone mass of the spine, hip, and total body. The selective estrogen receptor modulator (SERM) raloxifene binds to the estrogen receptor and is approved for the prevention and treatment of osteoporosis as well as the prevention of breast cancer. Tamoxifen is another well-known SERM, but it is only approved for the treatment and prevention of breast cancer. Both drugs have a favorable effect on bone turnover and bone mass. Raloxifene has been demonstrated in clinical trials to reduce the occurrence of vertebral fracture by 30%–50%, although the effect on nonvertebral fracture is not known. Bisphosphonates are the most widely used category of medications for the prevention and treatment of osteoporosis. Alendronate, risedronate, ibandronate, and zoledronic acid are approved medications in this class. Bisphosphonates act to impair osteoclast function and reduce osteoclast number by inducing apoptosis. Zoledronic acid is retained in the bone for a very long time and is dosed intravenously only once yearly. Exogenous administration of calcitonin, a polypeptide hormone produced by the thyroid gland, is sometimes administered as a nasal spray in the treatment of osteoporosis. It acts to suppress osteoclast activity by direct action on the osteoclast calcitonin receptor. In clinical studies, the effect on bone mass and vertebral fracture risk was small, and there was no effect on nonvertebral fractures. Denosumab is a fully human monoclonal antibody to RANKL, the final common effector of osteoclast formation, activity, and survival. When denosumab binds to RANKL, osteoclast maturation is significantly impaired. It is administered by subcutaneous injection twice yearly and has been demonstrated to decrease fracture risk in the spine, hip, and forearm over a 3-year period by 20%–70%. Teriparatide is a recombinant PTH (1-34hPTH) that is approved for the treatment of osteoporosis. It is administered by daily subcutaneous injection and has been shown to decrease both vertebral and nonvertebral fracture risk. Because teriparatide is an analogue of PTH, the drug acts like PTH with direct actions on osteoblast to stimulate new bone formation, which is unique among the treatments for osteoporosis.
++
++
++
A 38-year-old woman with cystic fibrosis and vitamin D deficiency has a T-score of –2.8 in the lumbar spine and hip. She is initiated on treatment with alendronate 70 mg weekly, cholecalciferol 5000 units daily, and calcium carbonate 1500 mg daily. When should the bone densitometry testing be repeated to assess the response to therapy?
++
++
++
++
++
++
++
++
++
++
++
E. It does not need to be repeated. MRI should be performed instead.
+
++
The answer is B. (Chap. 425) This individual with cystic fibrosis has malabsorption of vitamin D and chronic inflammation, placing her at increased risk of osteoporosis. Upon diagnosis of osteoporosis with a T-score less than –2.5, the patient was appropriately initiated on therapy with a bisphosphonate, vitamin D3, and calcium. The appropriate interval for following osteoporosis with bone densitometry after initiating treatment is not clearly established because most treatments yield only small or moderate bone mass increments. Thus, the changes need to be greater than approximately 4% in the spine and approximately 6% in the hip to be considered significant in any given individual. Medications take several years to produce significant changes in bone mineral density (BMD); therefore, bone densitometry should be repeated at intervals of >2 years. Only further declines in BMD should prompt a change in regimen.
++
++
++
A 19-year-old woman is evaluated by her primary care physician for recurrent long bone fractures. She has fractured her femur twice and her humerus three times. She has not had an abnormal number of falls and reports also having easy bruising. Aside from these repeated orthopedic injuries, she is otherwise healthy. Physical examination shows mildly disfigured bones, small, amber-yellowish teeth, and bluish-colored sclera. Osteogenesis imperfecta is suspected. Which of the following statements is true regarding this condition?
++
++
++
A. A mutation in type 1 procollagen likely is present in this patient.
++
++
B. Bone biopsy is needed for definitive diagnosis.
++
++
C. Bisphosphonates have shown long-term success in preventing long bone fractures in this condition.
++
++
D. Fractures in females tend to increase after puberty.
++
++
E. Increased bone mineral density may be demonstrated on x-ray absorptiometry.
+
++
The answer is A. (Chap. 427) Osteogenesis imperfecta (OI) is a heritable disorder of connective tissue in which there is a severe decrease of bone mass that makes bone brittle and prone to fracture due to a deficiency or abnormality in type I procollagen. The disease is often inherited in an autosomal dominant fashion. There are several subtypes of OI that are currently based on the clinical phenotype of the disease. There is debate about whether the disease should be reclassified based on genetic abnormalities, but at present, the classification based on clinical presentation remains the standard. Type 1 OI has a varied clinical presentation, but generally has the mildest bone disease with minimal or no apparent skeletal deformities. The disease may not present until adulthood in those with type 1 OI. However, type 2 OI produces very brittle bones and typically is lethal in utero or shortly after birth. Other types of OI have variable bone disease that can yield bone deformity with frequent fractures or kyphoscoliosis or result in only mild disease. Another common clinical feature of type 1 OI includes blue sclera, which is thought to be due to the thinness of the collagen fibers of the sclerae, allowing the choroid layers to be seen. Additionally, the teeth may have an amber or yellowish brown color due to a deficiency of dentin that is rich in type I collagen. The deciduous teeth are often smaller than normal, whereas the permanent teeth may be bell-shaped and restricted at the base. Hearing loss is common beginning in the second decade of life and affects >50% of individuals over the age of 30. Fractures tend to decrease after puberty in both sexes but may increase in women at the time of pregnancy and after menopause. Diagnosis of OI is usually based on clinical criteria in an individual with fractures and other typical clinical features. Given the autosomal dominant nature of inheritance, a family history of disease may be present. Decreased bone mineral density is demonstrated in a variety of imaging techniques including x-ray absorptiometry and plain radiographs. Bone biopsy is not required for diagnosis and may cause morbidity. Treatment of the disease is primarily aimed at treating complications. Fractures typically are only slightly displaced with little soft tissue swelling. Minimal support and traction are required. Although bisphosphonates are well tolerated and often used for moderate to severe disease, where they may decrease bony pain and fracture risk, their long-term effects and safety in osteogenesis imperfecta are unknown.
++
++
++
A 20-year-old man is evaluated during a routine physical examination prior to playing for a college basketball team. He was recruited to play on the team and offered a scholarship after being noticed for his skills on a junior national team abroad. He is originally from Nigeria and has come to the United States only for his education. His medical history is significant for prior treatment for tuberculosis at the age of 13. He takes no medications and has no allergies. His father died of sudden cardiac death at the age of 46. No autopsy was performed. Other family members in his father’s family have died at young ages from cardiac conditions. His mother is healthy. His height is 79 in (200.6 cm). His weight is 198 lb (89.8 kg). His BMI is 22.3 kg/m2. You note that his torso is short relative to his limbs. His arm span measures 83 in. He also has pectus excavatum and arachnodactyly. A high-arched palate is present. He wears glasses for severe myopia and has had ectopia lentis on the right. On cardiovascular examination, a II/VI blowing diastolic murmur is noted in the third left intercostal space. He is anxious to begin practicing with the basketball team. What do you advise at this time?
++
++
++
A. He is not safe for further competitive basketball or other strenuous physical activities.
++
++
B. He is safe to resume physical activity without further evaluation.
++
++
C. He may continue to practice with the team while further evaluation with an echocardiogram, slit-lamp examination, and genetic testing is performed.
++
++
D. He should be placed on a β-blocker and then can resume physical activity.
+
++
The answer is A. (Chap. 427, http://www.marfan.org/dx/home) This patient presents with evidence of Marfan syndrome (MFS), an autosomal disorder most commonly associated with mutations in the fibrillin-1 gene. MFS is one of the most common heritable connective tissue disorders, with an incidence of 1 in 3000–5000 and is found in most racial and ethnic groups. The diagnosis of MFS is based on the revised Ghent criteria, which include major and minor criteria for evaluation. In the most recent revision, there has been stronger emphasis on the cardiovascular and ocular manifestations of MFS. In the absence of family history, the presence of aortic root aneurysm and ectopia lentis is sufficient to make the diagnosis. The diagnosis can also be made with a combination of systemic manifestations and the presence of aortic root dilation or ectopia lentis. Some of the systemic manifestations that this patient exhibits include the presence of long limbs and tall stature. The ratio of the upper segment to the lower segment of the body is usually 2 standard deviations below mean for age, race, and sex, and the arm span is usually >1.05 times the height. Arachnodactyly with long, slender fingers and hands is present. Other skeletal deformities include pectus excavatum, pectus carinatum, scoliosis, kyphosis, pes planus, and high-arched palate. A calculator is available at http://www.marfan.org/dx/score to allow one to easily calculate the number of systemic symptoms an individual has on presentation. In individuals with a family history of MFS, the presence of ectopia lentis, aortic root dilation, or a positive systemic score would be adequate for diagnosis. In this clinical scenario, the patient has a strong likelihood of having MFS and should be advised to refrain from engaging in strenuous physical activity or contact sports immediately, although further workup with echocardiogram and slit-lamp examination will be required. Although there is no definitive family diagnosis, the sudden death of the patient’s father is likely to represent an aortic aneurysm rupture. In the most recent diagnostic criteria, an echocardiogram would be required for definitive diagnosis to evaluate for aortic root dilation. However, because this patient has several clinical features and a murmur concerning for aortic regurgitation, an echocardiogram would not be required before acting in the best interest of the man’s health and removing him from further physical activity. Cardiovascular abnormalities may include mitral valve prolapse with or without mitral regurgitation and aortic root dilation. Dilation of the aortic root and the sinuses of Valsalva are characteristic of MFS and are an ominous sign of the disease. The dilation can occur at any age and place the patient at risk for aortic regurgitation, aortic dissection, and aneurysmal rupture. Dilation can be accelerated by physical and emotional stress and pregnancy. Individuals may require surgical repair of the dilated aortic root, and routine follow-up echocardiography is required to ensure that further dilation is not occurring. Use of β-blockers and, more recently, angiotensin II receptor blockers has been demonstrated to reduce the rate or delay the onset of aortic dilation. Physical activity guidelines have been published that suggest that all patients with MFS should avoid strenuous physical activity and contact sports. However, regular exercise that is low impact and low intensity should be encouraged.
++
++
++
A 40-year-old man is evaluated as part of an executive physical examination. He has read about different screening procedures on the Internet and is interested in being screened for hemochromatosis. He is otherwise healthy and takes only a daily multivitamin. His father died of cirrhosis at the age of 56 and also drank alcohol heavily. There is no other liver disease in his family. Which of the following tests is the most appropriate first step to screen for this disorder?
++
++
++
A. Genetic testing for C282Y mutation
++
++
++
++
++
++
D. Screening for hemochromatosis is not cost effective and not advised
++
++
E. Transferrin saturation and serum ferritin
+
++
The answer is E. (Chap. 428) Hereditary hemochromatosis is a common genetic condition. One out of 10 individuals of northern European ancestry will be a heterozygous carrier for the most common mutation, HFE, and 0.3%–0.5% of the population are homozygous for this mutation. However, disease expression in individuals who are homozygous for the HFE gene varies widely and is modified by a variety of environmental and clinical factors including alcohol intake, dietary iron intake, blood loss from pregnancy and menstruation, and blood donation. It is estimated that about 30% of men who are homozygous for the HFE gene will develop symptomatic iron overload, with about 6% progressing to hepatic cirrhosis. In women, clinical disease is less prevalent, with only 1% progressing to cirrhosis. Clinical manifestations include iron overload (as measured biochemically) initially without symptoms, and then iron overload with symptoms. Initial symptoms often include lethargy, arthralgia, change in skin color, loss of libido, and diabetes mellitus. Cirrhosis, cardiac arrhythmias, and infiltrative cardiomyopathy are later manifestations. Because the clinical manifestations of the disease can be prevented with iron chelation and the mutation is so common, some have advocated for screening the population for evidence of iron overload. Although routine screening remains controversial, recent studies indicate that it is highly effective for primary care physicians to screen subjects using transferrin saturation and serum ferritin levels. This will detect anemia and iron deficiency as well. Liver biopsy or MRI may demonstrate later findings of increased iron deposition and/or cirrhosis, but these are more costly, possibly invasive, or risky and not recommended for screening. Genetic testing is also not recommended as a first step, although it is indicated if evidence of iron overload is found on serum iron studies. No HFE activity assay is currently available.
++
++
++
A 55-year-old white male with a history of diabetes presents to your office with complaints of generalized weakness, weight loss, nonspecific diffuse abdominal pain, and erectile dysfunction. The patient has a past history of hypercholesterolemia and takes atorvastatin. The examination is significant for hepatomegaly without tenderness, testicular atrophy, and gynecomastia. Skin examination shows a diffuse slate-gray hue slightly more pronounced on the face and neck. Joint examination shows mild swelling of the second and third metacarpophalangeal joints on the right hand. Which of the following studies is most likely to lead to the correct diagnosis?
++
++
++
A. Anti–smooth muscle antibody
++
++
++
++
C. Hepatic ultrasound with Doppler imaging
++
++
D. Hepatitis B surface antibody
++
++
E. HFE gene mutation screen
+
++
The answer is E. (Chap. 428) This patient presents with the classic finding of diffuse organ iron infiltration due to hemochromatosis. The iron accumulation in the pancreas, testes, liver, joints, and skin explains his findings. Hemochromatosis is a common disorder of iron storage in which inappropriate increases in intestinal iron absorption result in excessive deposition in multiple organs, but predominantly in the liver. There are two forms: hereditary hemochromatosis, in which the majority of cases are associated with mutations of the HFE gene, and secondary iron overload, which usually is associated with iron-loading anemias such as thalassemia and sideroblastic anemia. In this case, without a history of prior hematologic disease, the most likely diagnosis is hereditary hemochromatosis. Serum ferritin testing and plasma iron studies can be very suggestive of the diagnosis, with the ferritin often >500 μg/L and transferrin saturation of 50%–100%. However, these tests are not conclusive, and further testing is still required for the diagnosis. Although liver biopsy and evaluation for iron deposition or a hepatic iron index [(μg/g dry weight)/56 × age >2] provide the definitive diagnosis, genetic testing is widely available today is recommended for diagnostic evaluation because of the high prevalence of HFE gene mutations associated with hereditary hemochromatosis. If the genetic testing is inconclusive, an invasive liver biopsy evaluation may be indicated. Anti–smooth muscle antibody testing is useful for the evaluation of autoimmune hepatitis and is indicated in any case of cryptogenic cirrhosis. Plasma ceruloplasmin is the initial study in the evaluation of Wilson disease, which is also a cause of occult liver disease. However, Wilson disease would not be likely to be associated with pancreatic, joint, and skin findings. If chronic hepatitis B is suspected, a viral load or surface antigen test would be indicated. Hepatitis B surface antibody is useful to demonstrate resolved hepatitis B or prior vaccination. Hepatic ultrasound is useful in the evaluation of acute and chronic liver disease to demonstrate portal flow or vascular occlusion; it may be useful in the physiologic evaluation of this patient but would have little diagnostic value.
++
++
++
A 28-year-old man is admitted to the intensive care unit with fulminant hepatic failure and hemolysis. On further questioning, his family reports that he has been diagnosed with depression for 5 years and had a prior episode of acute hepatitis 2 years ago that resolved. At that time, his aspartate aminotransferase peaked at 1200 U/L and alanine aminotransferase peaked at 1900 U/L. He had only mild jaundice, with a total bilirubin of 7.2 g/dL. No cause of the hepatitis was found despite a workup that included viral and autoimmune causes. His liver function returned to normal. He is taking an antidepressant and occasional ibuprofen, but no other medications. Physical examination is notable for ascites and altered mental status with dystonia. Abdominal CT scan shows no biliary obstruction but a cirrhotic liver. Which of the following findings would be most likely to confirm the underlying diagnosis?
++
++
++
A. 24-Hour urine level of iron
++
++
B. Brain MRI showing damage to the basal ganglia
++
++
C. Genotype for HFE mutation
++
++
D. Schistocytes on peripheral blood smear
++
++
E. Slit-lamp ocular examination showing Kayser-Fleischer rings
+
++
The answer is E. (Chap. 429) This patient presents with liver disease, hemolysis, and psychiatric illness, which suggests the presence of Wilson disease. Wilson disease is an autosomal recessive disorder caused by mutations in the ATP7B gene, a copper-transporting ATP-ase. As a result of this mutation, patients store abnormally high levels of copper in their liver initially, but later, copper is stored in other organs such as the brain. Although liver dysfunction is a hallmark of the disease, it may have several presentations, such as acute hepatitis, cirrhosis, or hepatic decompensation, as in this patient. Hemolysis may complicate acute decompensation because of massive release of copper from the liver into the blood, leading to hemolysis. Accumulation of copper in the basal ganglia results in Parkinson-like syndromes. Up to 50% of patients with Wilson disease will have Kayser-Fleischer rings on ocular slit-lamp examination. These brownish rings surrounding the cornea are due to copper deposition within the cornea and are diagnostic when found. Twenty-four-hour urinary copper levels are universally elevated in this disease and are the primary diagnostic modality when Kayser-Fleischer rings are absent. Liver biopsy can also be used to confirm increased copper content. Although MRI will show basal ganglia damage, it is not specific for Wilson disease. HFE mutation is present in hemochromatosis, which this patient does not have. Urine iron levels are not indicated.
++
Therapy for Wilson disease is dependent on the degree of disease at the time of presentation. Patients with mild hepatitis may be treated with zinc, which blocks intestinal absorption of copper, results in a negative copper balance, and induces hepatic metallothionein synthesis, which sequesters additional toxic copper. Trientine serves as a copper chelator and is used for more severe liver dysfunction or neurologic or psychiatric disease. In acutely decompensated liver failure, zinc should not be administered for at least 1 hour following trientine because the zinc could be chelated instead of copper if administered simultaneously. Liver transplantation is appropriate for patients who have experienced treatment failed with initial therapy.
++
++
++
A 28-year-old man is admitted to the intensive care unit with fulminant hepatic failure and hemolysis. On further questioning, his family reports that he has been diagnosed with depression for 5 years and had a prior episode of acute hepatitis 2 years ago that resolved. At that time, his aspartate aminotransferase peaked at 1200 U/L and alanine aminotransferase peaked at 1900 U/L. He had only mild jaundice, with a total bilirubin of 7.2 g/dL. No cause of the hepatitis was found despite a workup that included viral and autoimmune causes. His liver function returned to normal. He is taking an antidepressant and occasional ibuprofen, but no other medications. Physical examination is notable for ascites and altered mental status with dystonia. Abdominal CT scan shows no biliary obstruction but a cirrhotic liver. Which of the following is the most appropriate initial treatment for the patient described?
++
++
++
++
++
++
++
++
++
++
++
+
++
The answer is D. (Chap. 429) This patient presents with liver disease, hemolysis, and psychiatric illness, which suggests the presence of Wilson disease. Wilson disease is an autosomal recessive disorder caused by mutations in the ATP7B gene, a copper-transporting ATP-ase. As a result of this mutation, patients store abnormally high levels of copper in their liver initially, but later, copper is stored in other organs such as the brain. Although liver dysfunction is a hallmark of the disease, it may have several presentations, such as acute hepatitis, cirrhosis, or hepatic decompensation, as in this patient. Hemolysis may complicate acute decompensation because of massive release of copper from the liver into the blood, leading to hemolysis. Accumulation of copper in the basal ganglia results in Parkinson-like syndromes. Up to 50% of patients with Wilson disease will have Kayser-Fleischer rings on ocular slit-lamp examination. These brownish rings surrounding the cornea are due to copper deposition within the cornea and are diagnostic when found. Twenty-four-hour urinary copper levels are universally elevated in this disease and are the primary diagnostic modality when Kayser-Fleischer rings are absent. Liver biopsy can also be used to confirm increased copper content. Although MRI will show basal ganglia damage, it is not specific for Wilson disease. HFE mutation is present in hemochromatosis, which this patient does not have. Urine iron levels are not indicated.
++
Therapy for Wilson disease is dependent on the degree of disease at the time of presentation. Patients with mild hepatitis may be treated with zinc, which blocks intestinal absorption of copper, results in a negative copper balance, and induces hepatic metallothionein synthesis, which sequesters additional toxic copper. Trientine serves as a copper chelator and is used for more severe liver dysfunction or neurologic or psychiatric disease. In acutely decompensated liver failure, zinc should not be administered for at least 1 hour following trientine because the zinc could be chelated instead of copper if administered simultaneously. Liver transplantation is appropriate for patients who have experienced treatment failed with initial therapy.
++
++
++
A 22-year-old woman presents to the emergency department for abdominal pain that she rates a 10 out of 10 in severity. Her current episode began about 5 hours ago. She describes it as diffuse and constant with a mild crampy quality. She has mild nausea and has vomited once. She feels that her abdomen is distended. Over the past 6 years, she has presented to the emergency department five times with similar symptoms and has been frustrated because nothing has been found. She reports that she was generally treated with IV fluids, antiemetics, and IV narcotics. She expresses that she was treated like a drug user because laboratory studies, urinalysis, and abdominal CT with IV and oral contrast were negative. Her symptoms have always resolved without intervention or hospitalization within 24–48 hours. She has had milder episodes approximately four to five times yearly and has avoided coming in for evaluation due to her prior bad experiences with healthcare. When she has pain, she has significant anxiety and insomnia. She has had two episodes of auditory hallucinations during an acute pain attack, which she attributed to the severity of the pain. She is currently in her senior year of college studying mechanical engineering and is an honors student. She has no past medical history. She stopped taking oral contraceptives due to her perception that it caused her episodes of abdominal pain to worsen. Her mother has had a few mild episodes of similar abdominal pain that she thinks has been due to endometriosis. Her mother has not sought specific evaluation. On examination, vital signs are heart rate of 120 bpm, temperature of 37.2°C, blood pressure of 138/88 mmHg, respiratory rate of 18 breaths/min, and arterial oxygen saturation (SaO2) of 99% on room air. She appears in mild distress due to pain. Examination of the head, eyes, ears, nose, and throat is unremarkable. Chest is clear. Cardiovascular exam shows only a regular tachycardia without murmur. Abdominal examination shows hypoactive bowel sounds with mild distention. There is no localizing tenderness. Abdominal x-rays show an ileus. A urinalysis and toxicology screen are negative. Her complete blood count and comprehensive metabolic panel are normal with the exception of mild hyponatremia (sodium 132 mmol/L). Which of the following is the next most appropriate step in her evaluation?
++
++
++
A. Endoscopy and colonoscopy
++
++
B. Plasma HMB synthase mutation analysis
++
++
C. Measurement of urine porphyrobilinogen and 5-aminolevulinic acid during attack
++
++
D. Measurement of urine porphyrins
++
++
E. Prescription for lubiprostone
+
++
The answer is C. (Chap. 430) This patient has a classic presentation for acute intermittent porphyria (AIP), an inherited disorder of heme biosynthesis. There are many different types of porphyrias, which are classified as either hepatic or erythropoietic depending on the primary site of overproduction and accumulation of their respective porphyrin precursors or porphyrins. AIP is classified as a hepatic porphyria and is typically inherited in an autosomal dominant fashion, although clinical expression of the disease is variable. It is most common in Scandinavia and Great Britain, with an estimated frequency of approximately 1 in 20,000. The genetic defect in AIP occurs in the enzyme hydroxymethylbilane (HMB) synthase, and over 300 mutations in this enzyme have been described. The disease typically presents with attacks of acute abdominal pain and neurologic symptoms that occur after puberty. Often, a precipitating cause of symptomatic episodes can be identified such as steroid hormone use, oral contraceptive use, systemic illness, reduced caloric intake, or use of many other medications (Table X-99A) This diagnosis should be considered in any individual with recurrent abdominal pain, especially when accompanied by neuropsychiatric complaints. The abdominal symptoms are often more prominent and nonspecific. These include severe nonlocalizing abdominal pain, abdominal distention, constipation or diarrhea, and vomiting. However, the physical examination shows no localizing findings, and workup fails to show any abnormalities with the exception of mild ileus. Other common findings on examination include tachycardia and hypertension due to increased sympathetic activation. Fever and leukocytosis are rare. Neuropsychiatric findings are considered a part of the diagnostic features and can be quite variable. A peripheral motor neuropathy that is associated with motor weakness and absent reflexes may occur. Sensory changes are less prominent. Psychiatric features may include depression, anxiety, insomnia, and hallucinations. The initial diagnostic test of choice for an acute porphyria is to perform a spot urine for presence of urinary porphyrin precursors (urinary porphyrobilinogen and 5-aminolevulinic acid) during an attack. Urinary porphyrobilinogen (PBG) is almost always elevated during an attack of AIP or one of the other acute porphyrias but not in any other medical condition, making it sensitive and specific for diagnosis of an acute porphyria (Table X-99B). A 24-hour collection is not required and may only delay diagnosis. If the urinary PBG is elevated, then second-line testing is measurement of red blood cell HMB synthase levels and urinary, plasma, and fecal porphyrins. Urinary porphyrin levels are not recommended as a screening test, however, because they are not sensitive markers. Other conditions, including chronic liver disease, can cause elevations in urinary porphyrins. If the diagnosis of AIP is made, genetic analysis for mutations in the HMB synthase gene should be performed, but this is not the test of choice for initial screening and diagnosis. The porphobilinogen level will drop in the recovery phase and may be normal between attacks. Therapy for acute attack is with carbohydrate loading, narcotic pain control, anxiolysis, and IV hemin, which repletes the end product in heme synthesis.
++